You are on page 1of 173

Ophthalmology MCQs ……………………………………………………….….

Tanta university

Ophthalmology MCQs

Questions Bank

With answer

1
Ophthalmology MCQs ……………………………………………………….…. Tanta university

CONJUNCTIVA

Dr Osama shalaby

1- Conjunctival injection is characterized by the following except:

a-Bright red colour.

b-Movable.

c-Not affected by vasoconstrictors.

d-Individual vessels are easily distinguished.

2- One of these is not manifested by ciliary injection:

a-Corneal ulcer

b-Viral conjunctivitis.

c-Acute congestive glaucoma.

d-Acute iridocyclitis.

3- Persistent unilateral conjunctivitis is usually due to:

a-Purulent conjunctivitis.

b-Chronic dacryocystitis.

c-Mucopurulent conjunctivitis.

d-Foreign body.

4- In ophthalmia neonatorum, all are true except:

a-Caused by birth trauma.

b-Frequently caused by gonococcal infection.


c-Maternal infection plays a role.

d-Silver nitrate drops were used as a prophylaxis.

5- All the following can be caused by chlamydial infection except:

a-Ophthalmia neonatorum

b-Trachoma.

c-Inclusion Conjunctivitis.

d-Central corneal ulcer.

6- These organisms can be seen normally in the conjunctiva:

a-Koch- Weeks bacillus.

b-Pneumococci.

c-Corynobacterium xerosis.

d-Corynobacterium diphtheria.

7- Most common organism in purulent conjunctivitis is:

a-Pneumococci.

b-Streptococci.

2
c-Gonococci.

d-Herpes simplex virus.

8- Subconjunctival hemorrhage is not caused by:

a-Trauma.

b-Mucopurulent conjunctivitis.

c-Adenoviral infection.

d-Acute hemorrhagic conjunctivitis.

9- Which is true about vernal conjunctivitis :

a-Always unilateral.

b-Usually occurs in young boys.

c-Antibiotic drops are the main therapy.

d-Main symptom is foreign body sensation.

10- Patient presented with itching, lacrimation, excoriation and macerated outer

canthus, the claimed organism is:

a-Morax Axenfeld diplobacillus.

b-Haemophylus influenza.

c-Pnumococci..

d-Koch- Weeks diplobacillus.

11- Old asthmatic hypertensive patient, presented with severe red eye after acute

attack of cough, most propably may be due to:

a-Corneal abrasion.
b-Acute conjunctivitis.

c-Spontaneous subconjunctival hemorrhage.

d-Acute iritis.

12- All are sure signs of trachoma except:

a-Arlt’s line.

b-Papillae of upper tarsal conjunctiva.

c-Herbert’s Pits.

d-Expressible follicles.

13- Itching is common with:

a-Spring catarrh.

b-Trachoma.

c-Mucopurulrnt conjunctivitis.

d-Corneal ulcer.

14- The secretions of spring catarrh are rich in:

3
a-Eosinophils.

b-Neutrophils.

c-Basophils.

d-Lymphocytes.

15- Pinguecula is:

a-Fatty degeneration.

b-Hyaline degeneration.

c-Elastoid hyaline degeneration.

d-Elastoid degeneration.

16- Giant papillary conjunctivitis can be caused by the following except:

a-Artificial prosthesis.

b-Spring catarrh

c-Contact lens wear.

d-Acute conjunctivitis.

17- Topical treatment used for phlyctenular conjunctivitis is:

a-Antibiotic drops.

b-Vasoconstrictor drops.

c-Corticosteroid drops.

d-Antiviral drops.

18- These may cause pterygium, except:

a-Exposure to ultra violet rays.


b-Viral infection.

c-Pinguecula.

d-Living in tropical area.

19- Patient had a pterygium, excised since one month, and starts to see double

vision, this may be due to:

a-Medial rectus weakness.

b-Lateral rectus paralysis.

c-Symblepharon formation

d-Recurrence.

20- These treatments are useful in preventing the recurrence after pterygium

excision except:

a-Topical antibiotics.

b-Topical corticosteroids.

c-Beta irradiation.

4
d-5 FU eye drops.

21- Which of the following is specific for the diagnosis of allergic conjunctivitis?

a-Eye redness

b-Itching

c-Foreign body sensation

d-Excessive lacrimation

22-A patient suffered from acute onset of facial palsy, the first line of treatment is:

a-Frequent ocular lubrication.

b-Lateral tarsorrhaphy

c-Topical corticosteroids.

d-Levator muscle resection.

23- The levator palpebrae superioris is inserted into the following structures except:

a-Skin of upper eye lid

b-Upper border of tarsus

c-Bulbar conjunctiva

d-Medial orbital margin & medial palpebral ligament

24- Rolling in of the lower lid margin can be due to:

a-Thermal injury of lid skin

b-Facial palsy

c-Trachoma
d-Ophthalmoplegia

25- The most important examination in case of congenital ptosis is:

a-The state of extraocular muscles

b-Fundus examination

c-Amount of levator function

d-Pupillary light reflex

26- Stye is an acute suppurative inflammation of:

a-Meibomian glands

b-Accessory lacrimal glands

c-Zeiss glands of the lash follicles

5
d-Lid margin

27- A female patient C / O diffuse hyperemic lid margin with multiple grayish

yellow crustations covering the lashes. The best treatment is:

a-Epilation of affected lashes

b-Electrolysis

c-Hot fomentations and local antibiotics

d-Systemic corticosteroids

28- In recurrent squamous blepharitis you should:

a-Give long acting corticosteroids

b-Give long acting antibiotics

c-Correct any refractive errors

d-Give maintenance dose of vitamins.

29- Chalazion is defined as:

a-Acute suppurative inflammation of meibomian glands

b-Chronic suppurative inflammation of meibomian glands

c-Chronic inflammatory lipogranuloma of meibomian glands.

d-Chronic non granulomatous inflammation of meibomian glands

30- Chalazion can cause the following complications except:

a-Irrigular astigmatism

b-Mechanical ptosis

c-Anterior uveitis
d-Internal hordeolum

31- A male patient is C / O chronic eye lid redness and frequent loss of lashes. The

most propable diagnosis is

a-Cicatricial entropion

b-Squamous blepharitis

c-Ulcerative blepharitis

d-Active trachoma

32- A case presented with hypermic lid margin, matting of eye lashes, yellow

crustations. The treatment include all the following except:

a-Local lid hygeine

b-Rubbing the lid margin by antibiotic ointment

c-Elctrolysis

d-Systemic antibiotic

33- Epilation of maldirected lashes is indicated in:


a-When the number is less than four

b-When the lashes are close together

c-In presence of acute corneal ulcer

d-In cases of high refractive error.

34- Congenital ptosis may be associated with the following congenital anomalies

except:

a-Blepharophimosis

b-Telecanthus

c-Epicanthus

d-Naso lacrimal duct obstruction.

35- Complications of congenital causes include the following except:

a-Ocular torticollis.

b-Amblyopia.

c-Complicated cataract.

d-Anbormal head posture.

36- Lagophthalmos can be caused by the following except:

a-Hyperthyroidism.

b-Facial palsy.

c-Severe entropion

d-Lid coloboma.

37- The commonest cause of bilateral ptosis is:


a-Horner syndrome.

b-Third nerve palsy.

c-Congenital

d-Mechanical.

38- Lid splitting and everting sutures is an operation used for the correction of:

a-Pure trichiasis of the upper eye lid.

b-Trichiasis and entropion of the upper eye lid.

c-Ectropion of the lower eye lid.

d-Paralytic entropion of the lower eye lid.

39- All these are true about ulcerative blepharitis except:

a-Can cause madarosis.

b-Can be complicated by ulcerative keratitis.

c-Can be caused by Morax Axenfeld bacillus.

d-Can be treated by antibiotics.

7
40- A 65 ys old patient had recurrence of chalazion after removal from the same

site two times. The best management is:

a-Systemic antibiotic and steroids.

b-Excision and histopathological evaluation.

c-Excision and curette evacuation.

d-Excision and cautery of the edges.

41- A patient has about 10 maldirected localized lashes of the upper eye lid. The

treatment of choice is:

a-Snellen’s operation.

b-Lid splitting and cryo application.

c-Epilation

d-Weiss procedure.

42- Fasaenella operation for ptosis is carried out in cases with:

a-Severe ptosis. Levator action less than 5 mm.

b-Moderate ptosis. Levator action 5-8 mm.

c-Mild ptosis. Levator action more than 8 mm.

d-None of above.

43- Incision and curette of chalazion should be.

a-Vertical.

b-Horizontal.

c-Any shape.

d-circular.
44- Grey line indicates a tissue plane between:

a-Skin muscle layer & tarsus conjunctival layer.

b-Tarsus & canjunctiva.

c-Skin & meibomian glands.

d-Palpebral conjunctiva & meibomian gland orifices.

45- Ankyloblepharon is :

a-The adhesion of the lids.

b-The adhesion between palpebral and bulbar canjunctiva.

c-The adhesion of the margins of the two lids.

d-All of the above.

46- Glands of Zeis are:

a-Modified sweat glands.

b-Modified sebaceous glands.

c-Modified meibomian glands.


d-None of above.

47- Levator palpebrae is inserted into:

a-Upper border of the tarsus.

b-Skin of upper lid.

c-Upper fornix.

d-All of above.

48- Hordeolum externum is an acute suppurative inflammation of:

a-Gland of Zeis.

b- Gland of Moll.

c- Gland of Wolfring.

d- Gland of Krause.

49- All of the following types of entropion are known except:

a-Spastic entropian.

b-Senile entropion.

c-Paralytic entropion.

d-Cicatricial entropion.

50- The amount of normal levator function is :

a-5 mm.

b-8 mm.

c-25 mm.

d-13 mm .
51- In brow suspension operation of ptosis, the best suspension material is :

a-Fascia lata.

b-Supramid.

c-Prolene.

d-Silicone.

53- All of the following are the causes of lagophthalmus except:

a-Facial nerve palsy.

b-Proptosis.

c-Lid fibrosis.

d- Third nerve paralysis.


54- A 30 ys old patient was subjected to face burn with strong acid, two months later

he presented with watering and inability to close his left eye. The explanation of this

may be:

a-Mechanical ectropion.

b-Cicatricial ectropion.

c-Paralytic ectropion.

d-Corneal ulcer.

55- A patient subjected to vertical lid wound, he is unable to to close his eye

properly. This condition can lead to:

a-Corneal scarring

b-Exposure keratopathy.

c-Vascularized corneal scar.

d-Corneal pannus.

56- The first line of treatment in acid burn of the eye is:

a-Eye patching.

b-Immediate wash with plain water.

c-Instilling local antibiotic drops.

d-Neutralization of the acid with alkali.

57- A 10 ys old boy, received blunt ocular trauma by tennis ball to his right eye, you

will expect to have:

a-Hypopion ulcer.
b-Blood staining of the cornea.

c-Hyphema.

d-Tractional retinal detachment.

58- A patient had penetrating eye injury in the right eye, the first aid management

is:

a-Washing with plain water.

b-Sterile eye bandage.

c-Application of antibiotic ointment.

d-Instilling atropine eye drops.

59- A patient is C / O monocular diplopia after blunt ocular trauma, the following

could cause this except:

a-Sublaxated lens.

b-Iridodialysis.

c-Traumatic hyphema.

d-Incipient immature cataract.


60- A patient had blunt ocular trauma, now he is C / O severe visual defect, the

cause of this may be due to:

a-Anteflexion of the pupil.

b-Berlin’s edema.

c-Conjunctival chemosis.

d-Angle recession.

61- Blunt ocular trauma commonly results in:

a-Blue dot cataract.

b-Anterior subcapsular cataract.

c-Posterior subcapsular cataract.

d-Coronary cataract.

62- A patient with a history of blunt trauma to the left eye C / O double vision that

disappears on covering the left eye & persists on covering the right eye.

Examination of this patient would reveal:

a-Miotic pupil.

b-Ectropion uveae.

c-Pupil showing lens equator.

d-Dilated pupil.

63- Trauma to the eye cannot cause:

a-Vitreous hemorrhage.

b-Macular edema.
c-Central retinal vein occlusion.

d-Retinal breaks.

64- A patient had blunt ocular trauma & C / O double vision that disappears on

covering either eye. The cause might be:

a-Orbital hematoma.

b-Corneal edema.

c-Orbital blow out fracture.

d-Iridodialysis.

65- A aptient had history of blunt ocular trauma 3 months ago, now is C / O severe

headache due to increased intraocular pressure.the most important diagnistic tool

is:

a-Automated field of vision .

b-Manual field of vision.

c-Gonioscopic examination.

d-Fundus examination.
66- A patient with recent history of ocular trauma & C/ O blurry vision.ocular

motility was normal, the most needed investigation is:

a-Ocular ultrasound.

b-Fluorescein angiography.

c-Field of vision.

d-Performing CT brain.

67- Etiology of sympathetic ophthalmia is:

a-Viral

b-Allergic

c-Bacterial

d-None

68- Prodromal symptoms of sympathetic ophthalmia is:

a-Pain

b-Redness

c-Photophobia

d-lacrimation

69- Sympathetic ophthalmia is rarely seen in:

a-Corneo scleral wounds

b-PECCE.

c-Acute suppuration

d-Iris encarceration.
70- Retained Intra Orbital FB may not be removed if

a-Sterile & inert

b-Mild visual affection

c-Its removal will affect the vision.

d-All of the above.

71- Metallic IOFB can be localized by the following methods except

a-Limbal ring & X ray

b-CT scan

c-US

d-MRI

72- Pathognomonic sign of IOFB

a-Corneal wound

b-Root in the iris

c-Traumatic cataract
d-hyphema

73- In siderosis bulbi, iron gets:

a-Deposited in membranes

b-Combined with cell proteins

c-Both

d-None

74- The following iris lesions caused by blunt trauma except:

a-Aniridia

b-Anteflexion

c-Retroflection

d-Heterochromia iridum

75- The weakest part of the eye affected by blunt trauma is:

a-Canal of Schlemm

b-Muscle insertion

c-Equator

d-Lens zonules

76- Blunt trauma coming down & out, the sclera ruptures:

a-Down & out.

b-Up & out.

c-up &in

d-Down & in.


77- Worker with arc light is exposed to:

a-UVR corneal burn.

b-Infra red heat burn.

c-Gamma radiation.

d-X ray radiation.

78- Which of the following conditions does NOT require emergency

ophthalmological management?

a-Anterior uveitis

b-Acute angle-closure glaucoma

c-Orbital floor fracture

d-Orbital cellulitis

79- Solar viewing during an eclipse can cause:

a-Corneal ulcer

b-Orbital cellulitis
c-Macular burn

d-Retinal tear

80- Patient had right maxillary tumours treated successfully with multiple doses of

radiotherapy, after that he noted dramatic decrease of visual acuity of the right eye,

the explanation of this may be due to:

a-Complicated cataract.

b-Anterior uveitis.

c-Central retinal vein thrombosis.

d-Acute congestive glaucoma.

81- Which of the following is not advised in the early management of a patient with

hyphema?

a-Admission to hospital.

b-Cycloplegics.

c-IOP lowering agents.

d-None of the above.

82-All of the following are true about the nodal point of the eye except:

a-Rays undergo refraction

b-Lies just anterior to the posterior pole of the lens

c-Optic and visual axis intersect at the nodal point

d-Optical center of the eye.


83-In Emmetropia parallel rays come to a focus:

a-Behind the retina.

b-Infront of the retina.

c-On the retina.

d-Might be all of the above.

84-The term Ametropia refers to:

a-Myopia

b-Hypermetropia

c-Asigmatism

d-All of the above.

85-All of the following is true about Angle alpha except:


a-Angle between optic and visual axes.

b-Always positive in myopia.

c-Lies at the nodal point of the eye.

d-More than 5 degrees in hypermetropia.

86-In hypermetropia parallel rays come to a focus:

a-On the retina

b-Behind the retina

c-Infront of the retina.

d-All of the above.

87-All of the following is true about hyperopic eye except:

a-Small axial length.

b-Deep anterior chamber.

c-Flat cornea

d-Positive angle alpha.

88-The term manifest hypermetropia means:

a-Hypermetropia without atropine.

b-Hypermetropia with atropine.

c-Hypermetropia with ciliary muscle paralysis.

d-Hypermetropia corrected by accommodation.

89-In myopia parallel rays come to a focus:

a-On the retina


b-Behind the retina

c-Infront of the retina.

d-All of the above.

90-All of the following is true about myopic eye except:

a-Small axial length.

b-Deep anterior chamber.

c-Steep cornea

d-High refractive power.

91-All of the following is true about progressive myopia except:

a-Starts at younger age.

b-Stabilizes around the age of 18 years old.

c-Reaches higher degrees.

d-Associated with retinal complications.

92-All of the following are types of myopia except:


A-Simple myopia.

b-Facultative myopia.

c-Congenital myopia.

d-Progressive myopia.

93-Progressive myopia might be associated with the following except:

a-Tigroid fundus.

b-Pseudo-papillitis.

c-Myopic crescent

d-Posterior staphyloma.

94-All of the following is true about astigmatism except:

a-Parallel rays come to a point focus on the retina.

b-Might be regular or irregular.

c-Might be congenital of postoperative.

d-All of the above.

95-Astigmatism with rule is:

a-Less common than against the rule.

b-Vertical meridian more curved.

c-Horizontal meridian more curved.

d-None of the above.

96-All of the following is true about irregular astigmatism except:

a-Could be corrected with glasses.


b-Could be corrected with rigid lenes.

c-Occurs with corneal opacities.

d-Occurs with keratoconus.

97-In glasses for simple regular astigmatism we use:

a-Sphero-cylindrical lenses.

b-Cylindrical lenses.

c-Convex lenses.

d-Concave lenses.

98-In glasses for compound regular astigmatism we use:

a-Sphero-cylindrical lenses.

b-Cylindrical lenses.

c-Convex lenses.

d-Concave lenses.

99-All of the following is correct about anisometropia except:


a-Difference in refraction between both eyes.

b-Best corrected with glasses.

c-Might lead to amblyopia.

d-Might be congenital.

100-The following are factors affecting the onset of presbyopia:

a-Age

b-Refractive error.

c-Preferred working distance.

d-All of the above.

101-Snow flakes cataract is found in cases with

a) gout

b) rheumatoid arthritis

c) diabetes mellitus

d) hypothyroidism

102-Bitemporal hemianopia is due to

a) cavernous sinus thrombosis

b) pituitary gland tumours

c) orbital apex syndrome

d) pineal body tumour

103-Hyperglycemia may cause

a) index hypermetropia
b) index myopia

c) axial hypermetropia

d) axial myopia

104-Bitot,s spots may be caused by

a) vernal keratoconjunctivitis

b) vit A deficiency

c) vit B deficiency

d) trachoma

105-Thyroid orbital myopathy affects


a) the lateral rectus then the superior rectus muscles

b) the inferior rectus then the medial rectus muscles

c) the medial rectus then the superior rectus muscles

107-Scleral show and upper lid retraction may be caused by

a) Horner syndrome

b) Hyperthyroidism b

c) entropion

108-Right optic nerve atrophy and left papilledema may be caused by

a) Right occipital lobe mass

b) Left occipital lobe mass

c) Right frontal lobe mass c

d) Left frontal lobe mass

Glucoma

Dr. Tarek Ragaey

109-Which of the following is TRUE concerning the intraocular pressure?

a. It varies during the day with a peak in the early morning

b. Normal value is 10-21 mmHg


c. It can be normal in patients with glaucoma E

d. A and B only

e. All of the above

110-Which of the following is not a sign of acute angle-closure glaucoma:

A. Constricted, rounded pupil

B. Shallow anterior chamber

C. Ciliary congestion

D. High intraocular pressure a

E. Corneal epithelial edema

111-An early sign of congenital glaucoma is :

18
Ophthalmology MCQs ……………………………………………………….…. Tanta university

A. Amblyopia

B. Corneal edema

C. Leukocoria b

D. Ptosis

112-All of the following are true concerning open angle glaucoma except:

A. IOP above 25 mm Hg.

B. Cup/disc ratio of 0.8

C. Visual fields are normal

D. Parents has glaucoma c

113-Secondary angle closure glaucoma may be associated with all of the following

except:

A. Intumescent cataract

B. Uveitis

C. Pigmentary glaucoma c

D. Anterior lens dislocation

114-The procedure of choice after medical control of acute angle closure glaucoma

is:

A. Laser iridotomy

B. Goniotomy a
C. cyclophotocoagulation

115-Pigmentary glaucoma is characterized by all of the following except:

A. Iris transillumination defect

B. Krukenberg spindle\

C. The anterior chamber angle is closed c

D. High Intraocular pressure

116-All of the following are signs of congenital glaucoma except:

A. The cornea is enlarged

B. Habb`s striae

C. The anterior chamber is shallow c

D. IOP is usually high

117-The color of the pupil in Acute congestive glaucoma is:

A. Yellow

B. Jet black

C. Grayish white E

19
Ophthalmology MCQs ……………………………………………………….…. Tanta university

D. Brown

E. Greenish blue

118-Neovascular glaucoma is due to all of the following except :

A. Proliferative Diabetic retinopathy

B. CRVO

C. Chronoic uveitis

D. Mature senile cataract

119-Goniotomy is Indicated in:

A. POAG after failure of medical treatment

B. Narrow angle glaucoma after failure of LPI C

C. Congenital glaucoma with clear cornea and diameter less than 13 mm.

D. Congenital glaucoma with corneal cloudiness.

120-Epiphora, photophobia and blepharospasm in a newly born infant occurs in:

A. Congenital naso-lacrimal duct obstruction.

B. Ocular inflammation (e.g. uveitis)

C. Corneal injury (e.g. abrasion) E

D. Congenital glaucoma

E. All of the above

121-All of the following are causes of lens induced glaucoma except:


A. lens-protein glaucoma (phacolytic, macrophage-induced )

B. Phacoanaphylactic

C. Intumescent cataract

D. Lens dislocation E

E. Nuclear sclerosis

123-Trabeculectomy is indicated in:

A. POAG after faliure of medical tretment

B. Narrow Angle Glaucoma after faliure of LPI D

C. Congenital glaucoma after failure of goniotomy and trabeculotomy

D. All of the above

124-RING scotoma is a characteristic field defect in:

A. Severe primary open angle glaucoma

B. Retinitis pigmentosa

C. Pitutary adenoma

D. All of the above E

E. A and B

21
Ophthalmology MCQs ……………………………………………………….…. Tanta university

125- All these drugs decrease aqueous production except:

A. Timolol

B. Prostaglandins B

D. Acetazolamide

E. NONSELECTIVE α -ADRENERGIC AGONISTS

126-Interventions to decrease the IOP in patients with acute angle closure glaucoma

include:

A. Acetazolamide

B. Mannitol

C. Laser peripheral iridotomy E

D. Pilocarpine eye drops

E. All of the above

***********************************************************************

CORNEA

Dr Waleed A. Allam

127- If the average radius of curvature of the anterior surface of the cornea is 8 mm,
a radius of 8.2 mm in all meridia results in:

a-Myopia.

b-Hypermetropia.

c-Simple myopic astigmatism. B

d-Compound hypermetropic astigmatism.

e-No changes.

……………………………………………………………………………………………...

128- The cornea represents …………. of the total diopteric power of the eye.

a-1/2

b-3/4

c-2/3

d-4/5

e-Non of the above.

…………………………………………………………………………………………

129- Descemet’s Membrane is characterized by:

a-Easily destroyed, and can regenerate.

b-Easily destroyed, and never regenerates. E

21
Ophthalmology MCQs ……………………………………………………….…. Tanta university

c-Resistant to destruction, and can regenerate.

d-Resistant to destruction, and never regenerates.

e-Non of the above.

……………………………………………………………………………………………

130- The substantia propria of the cornea proper receives its blood supply from:

a-Medial and lateral palpebral arteries.

b-Anterior ciliary arteries.

c-Circulus arteriosus major. E

d-Circulus arteriosus minor.

e-Non of the above.

……………………………………………………………………………………………..

131- the corneal touch reflex involves the following cranial nerves:

a-II and III.

b-II and IV.

c-V and III.

d-V and VI. E

e-V and VII.

……………………………………………………………………………………………..

132- The corneal light reflex depends on the following, except:

a-Healthy tear film.

b-The convex mirror property of the cornea.

c-Corneal nerve fibers are demyelinated. C

d-Intact corneal epithelium.

e-The corneal epithelium is non-keratinized.


……………………………………………………………………………………………..

133- Corneal transparency results from the following anatomical factors, except:

a-The cornea is devoid of blood vessels.

b-The cornea is devoid of lymphatic vessels. D

c-The corneal nerve fibers are demyelinated.

d-The corneal epithelium is non-keratinized simple columnar epithelium.

e-The stromal lamellae are regularly arranged.

……………………………………………………………………………………………..

134- The direct and immediate consequence of corneal endothelial injury is:

a-Corneal vascularization.

b-Corneal edema.

c-Corneal hyposthesia.

d-Corneal scarring. B

e-Corneal ulcer.

……………………………………………………………………………………………..

135- The following bacteria can invade an intact corneal epithelium:

a-Gonococci.

b-Pneumococci. A

22
Ophthalmology MCQs ……………………………………………………….…. Tanta university

C-Streptococci.

d-Staphylococci.

e-Pseudomonas.

……………………………………………………………………………………………..

136- The following fungus is able to invade an intact corneal epithelium:

a-Candida albicans.

b-Aspergillus niger.

c-Actinomyces bovis.

d-Rhizops nigricans. E

e-Non of the above.

……………………………………………………………………………………………..

137- Symptoms of corneal involvement include the following, except:

a-Lacrimation.

b-Pricking pain.

c-Blepharospasm. D

d-Corneal edema.

e-Photophobia.

138- Ciliary injection is characterized by the following, except:

a-Thick, dilated, tortuous blood vessels.

b-Involves the 4 mm circumlimbal area.

c-Violecious color.

d-Not constricted by adrenaline drops. A

e-Accompanies severe intra-ocular infections.

……………………………………………………………………………………………..
139- The following facts are true about hypopyon in a case of typical hypopyon

corneal ulcer, except:

a-Contains large molecular weight proteins.

b-Contains Pneumococci.

c-Contains fibrin.

d-May elevate the IOP. B

e-Indicates associated iridocyclitis.

……………………………………………………………………………………………..

140- A patient with central corneal nebula was able to see uncorrected vision of

6/36. If this is a central macula instead, the vision would be:

a-6/12

b-6/18

c-6/24

d-6/36 E

e-6/60

……………………………………………………………………………………………..

141- Leucoma non-adherent is one of the complications of perforating corneal ulcers

that may occur in:

23
Ophthalmology MCQs ……………………………………………………….…. Tanta university

a-Healed central Descematocele.

b-Small central perforation with no iris prolapse.

c-Central dendritic corneal ulcer. B

d-Dense central KPs.

e-Non of the above.

……………………………………………………………………………………………..

142- Fluorescein stain helps diagnose perforating corneal ulcers complicated with

fistula formation. Fluorescein will stain:

a-Floor of the ulcer.

b-Edges of the ulcer.

c-Aqueous humor.

d-Dead epithelium. C

e-Corneal stroma.

……………………………………………………………………………………………..

143- Non-specific treatment of corneal ulcers should include the following, except:

a-Mydriatic cycloplegic drops.

b-Pain medications.

c-Antibiotics.

d-Eye bandage. C

e-Heat application.

……………………………………………………………………………………………..

144- The following bacteria is responsible for typical hypopyon corneal ulcers:
a-Gonococci.

b-Pneumococci.

c-Streptococci.

d-Staphylococci. B

e-Pseudomonas.

……………………………………………………………………………………………..

145- Descematocele formation is uncommon with:

a-Gonococci.

b-Pneumococci.

c-Streptococci.

d-Staphylococci. B

e-Pseudomonas.

……………………………………………………………………………………………..

146- The following are primary non-infective corneal ulcers, except:

a-Atheromatous corneal ulcers.

b-Neuroparalytic corneal ulcers.

c-Phlyctenular corneal ulcers.

d-Keratomalacia. C

24
Ophthalmology MCQs ……………………………………………………….…. Tanta university

e-Mooren’s ulcers.

……………………………………………………………………………………………..

147- Pain is a constant feature in corneal ulcers. Pain is most in:

a-Typical hypopyon corneal ulcers.

b-Herpetic corneal ulcers.

c-Acanthameba corneal ulcers. C

d-Lagophthalmos corneal ulcers.

e-Neuroparalytic corneal ulcers.

……………………………………………………………………………………………..

148- Pain is a constant feature in corneal ulcers. Pain is least in:

a-Typical hypopyon corneal ulcers.

b-Herpetic corneal ulcers.

c-Acanthameba corneal ulcers.

d-Lagophthalmos corneal ulcers.

e-Neuroparalytic corneal ulcers. C

149- Corneal hyposthesia is one of the diagnostic signs of:

a-Herpetic corneal ulcers.

b-Typical hypopyon corneal ulcers.

c-Atypical hypopyon corneal ulcers. A

d-Acanthameba corneal ulcers.

e-Mooren’s ulcers.

……………………………………………………………………………………………..

150- Coagulated (pyramidal) hypopyon is a sign of:

a-Herpetic corneal ulcers.


b-Typical hypopyon corneal ulcers.

c-Atypical hypopyon corneal ulcers. C

d-Acanthameba corneal ulcers.

e-Mooren’s ulcers.

……………………………………………………………………………………………..

151- The following corneal ulcers usually start near the limbus, except:

a-Fascicular ulcer.

b-Typical trachomatous ulcer.

c-Typical hypopyon ulcer.

d-Mooren’s ulcer. C

e-Marginal ring ulcer.

……………………………………………………………………………………………..

152- Lagophthalmos corneal ulcers usually start at:

a-The lower third of the cornea.

b-The middle third of the cornea.

c-The upper third of the cornea. A

d-The center of the cornea.

e-Anywhere allover the cornea.

25
Ophthalmology MCQs ……………………………………………………….…. Tanta university

……………………………………………………………………………………………..

153- Typical trachomatous corneal ulcers usually start at:

a-The lower third of the cornea.

b-The middle third of the cornea.

c-The upper third of the cornea.

d-The center of the cornea. C

e-Anywhere allover the cornea.

……………………………………………………………………………………………..

154- Topical steroids are contraindicated in the following conditions, except:

a-Typical hypopyon corneal ulcers.

b-Fungal corneal ulcers.

c-Dendritic corneal ulcers.

d-Herpetic interstitial keratitis. D

e-Acanthameba corneal ulcers.

155- Non-inflammatory corneal ectasia include:

a-Keratectasia.

b-Keratoconus.

c-Keratomalacia. B

d-All of the above.

e-Non of the above.

……………………………………………………………………………………………..

156- Pseudocornea is a term used to describe:

a-Keratectasia.

b-Keratoconus.
c-Keratomalacia. D

d-Total anterior staphyloma.

e-Non of the above.

……………………………………………………………………………………………..

157- Typically, progressive keratoconus leads to the following, except:

a-Frequent change of glasses.

b-Progressive irregular astigmatism.

c-Progressive hypermetropia. C

d-Fleisher ring.

e-Vogt stria.

……………………………………………………………………………………………..

158- Keratoconus can be managed by the following, except:

a-Glasses.

b-Rigid contact lenses.

c-LASIK.

d-Intracorneal ring segments. C

e-Penetrating keratoplasty.

26
Ophthalmology MCQs ……………………………………………………….…. Tanta university

***********************************************************************

MCQs

Prof Dr. Ashraf EL-Desouky

Single answer pattern

True and false pattern

Take care of every word in the statement

159-The leading cause of preventable blindness worldwide;

• a-Senile cataract

• b-Age related macular degeneration E

• c-Glaucoma

• d-Diabetic retinopathy

• e-trachoma

160-The term “mature cataract” means

• a-A nuclear cataract present more than 10 years

• b-A posterior subcapsular cataract that reduces visual acuity to 6/60 or worse

• c-A cortical cataract that involves the entire cortex

• d-An anterior subcapsular cataract that causes capsular wrinkling.

161-Criteria of mature senile cataract

a-Visual acuity HM

b-Absent RR
c-Absent iris shadow D

d-All of the above

162-Topical steroids are contraindicated in corneal ulcers

except:

• a-Phlectenular fasicular ulcer

• b-Dendritic ulcer A

• c-Typical hypopyon ulcer

• d-Atypical hypopyon ulcer

163-The commonest sign of Graves’ disease

• a-Exophthalmos

• b-Lid retraction B

27
Ophthalmology MCQs ……………………………………………………….…. Tanta university

• c-Diplopia

• d-Conjunctival chemosis

164-Acute proptosis may be due to

• a-Trauma

• b-Orbital cellulitis

• c-Rhabdomyosarcoma D

• d-All of the above

165-Enophthalmos may be due to

• a-Trauma

• b-Cachexia

• c-Post radiotherapy

• d-Secondaries of breast scirrhus carcinoma E

• e-All of the above

166-The commonest cause for night blindness

• a-Congenital

• b-Vitamine A deficiency A

• c-Nuclear cataract

• d-Retinitis pigmentosa

• e-Liver diseases

167-The only staphyloma with normal IOP is

• a-Partial anterior staphyloma

• b-Ciliary staphyloma

• c-Intercalary staphyloma E

• d-Equatorial staphyloma
• e-Posterior staphyloma

168-Ectropion of the upper eyelid may be

a-Senile

b-Paralytic

c-Congenital D

d-Non of the above

169-Episcleritis is similar to phlycten

clinically but differs in being

28
Ophthalmology MCQs ……………………………………………………….…. Tanta university

a-tender

b-flat

c-Pigmented A

d-multiple

170-Pneumococci can cause

a-Acute dacryocystitis

b-Chronic dacryocystitis

c-Atypical hypopyon ulcer B

d-Ulcerative blepharitis

171-Staphyloococci can cause

a-Acute dacryocystitis

b-stye

c-Atypical hypopyon ulcer E

d-Ulcerative blepharitis

e-All of the above

172-Etiology of ptrygium

• a-Neoplastic

• b-Infection

• c-Inflammation D

• d-degenerative

173-Endogenous septic focus may cause All except :


• A-Phlycten B

• B-Hypopyon ulcer

• C-Iridocyclitis

• D-Metastatic endophthalmitis

174-Optic nerve head in glaucomatous optic atrophy has all

except;

• A-Large deep cup

• B-Interrupted retinal vessels C

• C-Waxy yellow colour

• D-Overhanging margins

29
Ophthalmology MCQs ……………………………………………………….…. Tanta university

175-Doctor sees nothing & patient sees nothing in

• A-Papillitis

• B-Papilloedema C

• C-Retrobulbar neuritis (toxic amblyopia)

• D-All of the above

176-Which of the following ttt is used for optic neuritis

• A-prednisolon

• B-Observation A

• C-Antibiotics

• D-Atropine

177-Papilloedema leads to:

• A-Rapid deterioration of vision

• B-Amaurosis Fugax

• C-Pain on eye movements B

• D-Early loss of color vision

178-Papilloedema leads to the following Feild changes:

• A-Nasal step

• B-Arcuate scotoma D

• C-Cocentric contraction of peripheral Feild

• D-Enlarged blind spot

179-All are correct except


Consecutive optic atrophy occurs in:

• A-Degenerative myopia D

• B-Chorioretinitis

• C-CRAO

• D-CRV thrombosis

180-Afferent pupillary defect occurs in: All except

• A-Papillitis B

• B-Hysteria

31
Ophthalmology MCQs ……………………………………………………….…. Tanta university

• C-Optic atrphy

• D-Retrobulbar neuritis

181-In a baby with watery eyes which may be the cause;

• A-Congenital NLD obstruction

• B-Buphthalmos

• C-Viral conjunctivitis E

• D-Corneal abrasion

• E-All of the above

182-Rapid painful loss of vision; in all except

• A-AACG

• B-Blunt trauma D

• C-Alkali burn

• D-CRAO

183-CILIARY INJECTION; In all except;

• A-AACG

• B-CORNEAL ULCEER

• C-ANTERIOR UVEITIS D

• D-EPISCLERITIS

184-All the following are signs of lens sublaxation

except;

• A-Phakodenesis
• B-Iridodnesis

• C-Irrigular anterior chamber D

• D-Intact all zonule

185-All of the following are lens induced glaucoma except;

• A-Phaco morphic glaucoma

• B-Phacoanaphylactic glaucoma D

• C-Phacolytic glaucoma

• D-Neovascular glaucoma

31
Ophthalmology MCQs ……………………………………………………….…. Tanta university

186-The eye more susceptible to AACG

• A-Hypermetropic eye

• B-Myopic eye A

• C-Astigmatic eye

• D-Aphakic eye

187-In an acute angle closure glaucoma’ the choice of surgery

is decided after

• A-Gonioscopic examination

• B-Fundus examination A

• C-Tonometry

• D-Visual field examination

188-All of the following are the characteristics of glaucomatus cup except

• A-Large deep cup

• B-Overhanging margins

• C-Retinal vessels appear broken at the margin D

• D-Lamina criprosa is not visible

189-Acetazolamide lowers IOP by

• A-Decreased aquous production

• B-Increased aquous drainage A

• C-Lower episcleral venous peressure

• D-All of above

190-B. Blockers lower IOP by

• A-Decreased aquous production


• B-Increased aquous drainage A

• C-Lower episcleral venous peressure

• D-All of above

191-Pre auricular lymphadenopathy

Occurs with the following conjunctivitis

• A-Vernal keratoconjunctivitis

• B-Phlyctenular keratoconjunctivitis C

• C-Viral conjunctivitis

32
Ophthalmology MCQs ……………………………………………………….…. Tanta university

• D-Angular conjunctivitis

192-Tremulous iris can be seen in:

• A-Aphakia

• B-Sublaxation of the lens E

• C-Hypermature cataract

• D-Posterior dislocation of the lens

• E-All of above

193-Posterior polar cataract markedly affects vision because:

• A-Its shadow lies on the macula B

• B-Close to the nodal point

• C-It matures early

• D-It blocks the pupillary area

194-Nuclear cataract changes the refraction of the eye into

• A-Myopia

• B-Hypermetropia A

• C-Astigmatism

• D-No change

195-Most common cause of diminution of vision after

phakoemulsificatio is

• A-Cystoid macular edema B

• B-Posterior capsule opacification


• C-Corneal decompansation

• D-Retinal detachment

196-The best treatment for Posterior capsule opacification

• A-Surgical excision B

• B-Laser opening

• C-Surgical polishing

• D-Leave alone

33
Ophthalmology MCQs ……………………………………………………….…. Tanta university

197-The type of laser used to treat Posterior capsule

opacification

• A-Yag laser

• B-Argon laser

• C-Diode laser A

• D-Excimer laser

198-All of the following types of entropion are known except

• A-Spastic entropion

• B-Senile entropion C

• C-Paralytic entropion

• D-Cicatricial entropion

199-All of the following are causes of lagophthalmos except

• A-Facial nerve palsy

• B-Proptosis

• C-Cicatricial ectropion

• D-Third nerve paralysis D

200-Corneal ulcers can occur with the following CN disorders

• A-VII CN palsy

• B-III CN affections

• C-VI CN paralysis A

• D-IV CN paralysis

201-Ectropion of the upper lid most commonly;


• A-Spastic ectropion

• B-Senile ectropion D

• C-Paralytic ectropion

• D-Cicatricial ectropion

201-Munson’ sign in:

A-Corneal fistula

B-Corneal dystrophy C

C-Keratoconus

34
Ophthalmology MCQs ……………………………………………………….…. Tanta university

D-Corneal facet

202-In corneal edema; all are true except;

A-There is increase in corneal diameter

B-There is increase in corneal thickness

C-Cloudy cornea A

D-Epthelial bullae

E-Predispose to Corneal vascularization

203-Corneal damage with trachoma is due to:

A-trichiasis

B-dryness

C-Lagophthalmos and exposure D

D-All of the above

204-Double staining pattern of the cornea is characteristic for:

A-Fungal corneal ulcer

B-Herpetic corneal ulcer

C-Exposure keratopathy B

D-Acanthaembic corneal ulcer

205-Corticosteroids is given in:

A-Bacterial corneal ulcer

B-Herpetic corneal ulcer C

C-Fasicular phlyctenular ulcer

D-Stromal fungal keratitis

206-Tarrsorraphy is essential in:


A-Bacterial corneal ulcer

B-Viral corneal ulcer C

C-Exposure keratopathy

D-Traumatic corneal ulcer

207-Blood staining of the cornea is due to:

A-Hyphema

B-Hyphema with rise of IOP

C-Corneal edema B

D-Corneal FB

208-The pupil in acute Ant. Uveitis is:

A-Constricted

35
Ophthalmology MCQs ……………………………………………………….…. Tanta university

B-Dilated A

C-Festooned

D-Vertically oval

209-Infective corneal ulcers include all except:

A-Bacterial corneal ulcer

B-Fungal corneal ulcer C

C-Mooren’s ulcer

D-Viral corneal ulcer

210-In treating bacterial corneal ulcer all are true except:

A-Antibiotics drops

B-Vitamin A,C D

C-Mydriatics and cycloplegics drops

D-Corticosteroids drops

211-All of the following are non-specific signs in

conjunctivitis except

A-Subconjunctival hemorrhage C

B-Papillae

C-Follicles

D-pseudomembranes
212-In buphthalmos we should exclude all of the following

• Except

A-Retinoblastoma

B-Megalocornea D

C-High myopia

D-Babies of diabetic mothers

213-In buphthalmos which of the following is a late

presentation

A-Lacrimation and sneezing B

36
Ophthalmology MCQs ……………………………………………………….…. Tanta university

B-Optic cupping

C-Enlarged hazy cornea

D-Flattened sublaxated lens

214-In a patient with HM vision, visual feild can be tested by;

A-Projection of light

B-Cofrentation test A

C-Automated perimetry

D-Bjerrum screen

215-Which of the following is not a test for visual feild

A-Projection of light

B-Cofrentation test

C-Automated perimetry E

D-Bjerrum screen

E-Percepton of light

216-Which of the following is not a test for visual feild

A-Projection of light

B-Cofrentation test

C-Automated perimetry E

D-Bjerrum screen

E-Percepton of light

217-Glaucoma inversus can occur in

A-Post sublaxated lens

B-Post dislocated lens D

C-Intumescent cataract
D-Anterior dislocated lens

Glaucoma Inversus

• Glaucoma inversus occurs with lens sublaxation or dislocaton caused

by pupillary block induced by anteriorly displaced lens or herniated

vitreous

218-TTT is directed at relieving the pupillary block.

• A-Iridectomy is indicated

• B-Cycloplegics are helpful

• C-IOP lowering medications are employed

• D-Miotics are not helpful

• E-Surgical lens removal may be indicated in some cases

37
Ophthalmology MCQs ……………………………………………………….…. Tanta university

219-Glaucoma inversus can be treated by:

A-Pilocarpine + anti-inflammatories

B-Pilocarpine + beta blockers C

C-Atropine

D-cyclocryotherapy

220-Phakomorphic Glaucoma is;

A-Induced by intumescent cataract

B-Induces pupillary block

C-A closed angle secondary glaucoma E

D-Urgent cataract extraction is indicated

E-All of the above

221-Amaurotic cat’s eye reflex,

In all except

A-Retinoblastoma

B-PHPV

C-Coat’s disease E

D-Toxocara

E-MM choroid

222-Third C nerve innervate all except

• A-Superior oblique muscle

• B-Levator palpebre muscle


• C-Inferior oblique muscle A

• D-Medial rectus muscle

223-Horner’s syndrome

A-Ptosis + myosis + enophthalmos + anhydrosis

B-Ptosis + mydriasis + enophthalmos + anhydrosis A

C-lagophthalmos + myosis + enophthalmos + anhydrosis

D-diplopia + myosis + enophthalmos + anhydrosis

224-Diplopia due to right 6 CN palsy

• A-Increases on looking to

• B-To the right

38
Ophthalmology MCQs ……………………………………………………….…. Tanta university

• C-To the left B

• D-Up

• E-down

225-Diplopia due to right 4 th CN palsy

• A-Disappear on covering ; D

• B-Right eye

• C-Left eye

• D-Either one

226-The commonest cause of crossed eyes in the first year of life

• A-Infantile esotropia

• B-Accomodative esotropia A

• C-Six CN palsy

• D-Duane’s syndrome

227-convergent squint may be due to:

• A-Infentile esotrpia

• B-Accommodative esotropia E

• C-Six N. palsy

• D-Graves’ disease

• E-All of the above

Glaucoma Inversus

Dr Ashraf EL-Desouky

Glaucoma Inversus

Glaucoma inversus occurs with lens sublaxation or dislocaton caused by

pupillary block induced by anteriorly displaced lens or herniated vitreous


TTT is directed at relieving the pupillary block.

Iridectomy is indicated

Cycloplegics are helpful

IOP lowering medications are employed

Miotics are not helpful

Surgical lens removal may be indicated in some cases

***************************************

MCQ model

Dr Ashraf EL-Desouky

MCQs

39
Ophthalmology MCQs ……………………………………………………….…. Tanta university

1.Doctor sees nothing& patient sees nothing in

a) Papillitis

b) Papilloedema

c) toxic amblyopia (retrobulbar neuritis) C

d) All of the above

e) Non of the above

2.Papilloedema lead to:

a) Rapid deterioration of vision

b) Primary optic atrophy

c) Pain on eye movements D

d) Enlarged blind spot

e) Colored halos

3. Afferent pupillary defect occurs in all except

a) Papillitis

b) Hysteria

c) Orbital abscess B

d) Optic atrphy

e) Retrobulbar neuritis

4. Convergent squint may be due to:

a) Infentile esotrpia

b) Accommodative esotropia E
c) Six N. palsy

d) Graves’ disease

e) All of the above

5.All the following are signs of lens sublaxation except;

a) Phakodenesis

b) Iridodnesis

c) Irrigular anterior chamber E

d) High astigmatic error

e) Intact all zonule

6.All of the following are symptoms of Retinal Detachment except;

a) Floaters

b) Flashes of light E

c) Failing vision

d) Field defect

e) Ciliary injection

7.In acute angle closure glaucoma’ the choice of surgery is decided after

a) Gonioscopic examination

b) Fundus examination A

c) Tonometry

d) Visual field examination

e) Dark room test

f)

8.Enophthalmos may be caused by


a) Blunt orbital trauma

b) Schirrus carcinoma of the breast metastasis E

c) Cachexia

d) Radiotherapy close to orbital region

e) all of the above

9.Beta Blockers lower IOP mainly by

41
Ophthalmology MCQs ……………………………………………………….…. Tanta university

a) Decreased aquous production

b) Increased aquous drainage

c) Lower episcleral venous peressure C

d) All of above

e) Increase suprachoroidal aquous outflow

10.Pre auricular lymphadenopathy occurs with the following conjunctivitis

a) Vernal keratoconjunctivitis

b) Phlyctenular keratoconjunctivitis D

c) Viral conjunctivitis

d) Angular conjunctivitis

e) Ocular cicatricial pemphigoid

11.All of the following are causes of lagophthalmos except

a) Facial nerve palsy

b) Proptosis

c) Cicatricial ectropion

d) Third nerve paralysis

e) Thyroid Associated orbitopathy

12.In corneal edema, all is true except

a) There is increase in corneal diameter


b) There is increase in corneal thickness

c) Cloudy cornea

d) Predispose to Corneal vascularization

e) Wrinkled Descmet’ membrane

Good Luck

***********************************************************************

Tanta University

Faculty of Medicine

Under graduate mid year exam. May, 2007

MCQs

All questions are to be attempted Time allowed one hour

Select only one answer

1.Doctor sees nothing& patient sees nothing in

f) Papillitis

g) Papilloedema

h) Retrobulbar neuritis (toxic amblyopia)

i) All of the above

2.Which of the following treatment is used for optic neuritis

a) Prednisolon I.V.

b) Observation

c) Pilocarpine 2% A

d) Atropine 1%

3.Papilloedema lead to:

f) Rapid deterioration of vision

g) Primary optic atrophy


h) Pain on eye movements D

i) Optic disc edema more than 3 D

4. Afferent pupillary defect occurs in all except

f) Papillitis

g) Hysteria

h) Optic atrphy G

i) Retrobulbar neuritis

5.Rapid painful loss of vision occurs in all except

41
Ophthalmology MCQs ……………………………………………………….…. Tanta university

a) Acute angle closure glaucoma

b) Blunt trauma D

c) Alkali burn

d) CRAO

6.All the following are signs of lens sublaxation except;

f) Phakodenesis

g) Iridodnesis

h) Irrigular anterior chamber I

i) Intact all zonule

7.All of the following are symptoms of Retinal Detachment except;

f) Musca Volitants

g) Flashes of light

h) Deterioration of vision I

i) Sever ocular pain

8.The eyes susceptible to angle closure glaucoma are:

a) Hypermetropic eye

b) Myopic eye

c) Astigmatic eye A

d) Pseudophakic eye

9.In acute angle closure glaucoma’ the choice of surgery is decided after

g) Gonioscopic examination

h) Fundus examination A

i) Tonometry
j) Visual field examination

10.All of the following are the characteristics of glaucomatus cup except

a) Large deep cup

b) Overhanging margins

c) Retinal vessels appear broken of at the margin D

d) Lamina criprosa is not visible

11.Enophthalmos may be caused by

f) Blunt orbital trauma

g) Schirrus carcinoma of the breast metastasis

h) cachexia

i) all of the above

12.Beta Blockers lower IOP mainly by

f) Decreased aquous production D

g) Increased aquous drainage

h) Lower episcleral venous peressure

i) All of above

13.Pre auricular lymphadenopathy occurs with the following conjunctivitis

f) Vernal keratoconjunctivitis

g) Phlyctenular keratoconjunctivitis

h) Viral conjunctivitis C

i) Angular conjunctivitis

14.Tremulous iris can be seen in:

a) Aphakia

b) Hypermature cataract D

c) Posterior dislocation of the lens

d) All of above

15.Posterior polar cataract markedly affects vision because:


a) Its shadow lies on the macula

b) Close to the nodal point B

c) It matures early

d) It blocks the pupillary area

16.Nuclear cataract changes the refraction of the eye into

a) Myopia

b) Hypermetropia A

c) Astigmatism

42
Ophthalmology MCQs ……………………………………………………….…. Tanta university

d) No change

17.Most common cause of diminution of vision after ECCE is

a) Cystoid macular edema

b) Posterior capsule opacification B

c) Corneal decompansation

d) Retinal detachment

18.The best treatment for Posterior Capsule Opacification

a) Surgical excision

b) Laser opening

c) Surgical polishing B

d) Leave alone

19.The type of laser used to treat Posterior Capsule Opacification

a) Yag laser

b) Argon laser

c) Diode laser

d) Excimer laser

20.All of the following types of entropion are known except

a) Spastic entropion

b) Senile entropion C

c) Paralytic entropion

d) Cicatricial entropion

21All of the following are causes of lagophthalmos except

f) Facial nerve palsy

g) Proptosis

h) Cicatricial ectropion D
i) Third nerve paralysis

22.Ectropion of the upper lid most commonly;

a) Spastic ectropion

b) Senile ectropion

c) Paralytic ectropion D

d) Cicatricial ectropion

23.Munson’ sign occurs in:

a) Corneal fistula

b) Corneal dystrophy C

c) Keratoconus

d) Corneal facet

24.In corneal edema, all is true except

f) There is increase in corneal diameter

g) There is increase in corneal thickness A

h) Cloudy cornea

i) Predispose to Corneal vascularization

25.Double staining pattern of the cornea is characteristic for:

a) Fungal corneal ulcer

b) Herpetic corneal ulcer B

c) Exposure keratopathy

d) Acanthaembic corneal ulcer

26.Topical Corticosteroids are contraindicated except in:

a) Bacterial corneal ulcer

b) Herpetic corneal ulcer C

c) Fasicular phlyctenular ulcer

d) Stromal fungal keratitis

27.Tarrsorraphy is essential in:


a) Bacterial corneal ulcer

b) Viral corneal ulcer

c) Exposure keratopathy C

d) Traumatic corneal ulcer

28.Blood staining of the cornea is due to:

a) Hyphema

b) Hyphema with rise of IOP

c) Corneal edema

43
Ophthalmology MCQs ……………………………………………………….…. Tanta university

d) Corneal FB

29 All are infective corneal ulcers except

a) Typical hypopion ulcer

b) Atypical hypopion ulcer C

c) Fasicular ulcer

d) Dendritic corneal ulcer

30. all the following muscles are supplied by Oculomotor nerve except:

a) Superior oblique muscle

b) Inferior oblique muscle A

c) Inferior rectus muscle

d) Levator palpebre muscle

Good Luck

MCQsmodel 2

Ophthalmologic manifestations of rheumatoid arthritis may include all of the following, except

A. Secondary Sjögren's syndrome with sicca complex

B. Scleritis

C. Episcleritis

D. Corneal melts

E. Ischemic optic atrophy


You examine a 2-week-old infant and find a dense congenital cataract in the left eye. By what age

should the patient be referred for surgical evaluation?

A) 2 weeks

B) 6 months

C) 12 months

D) 24 months

E) 36 months

Correct is A

What is the shape of pupil in acute iridocyclitis

a.oval and dilated

b.small and irregular

c.large and round

d.none

Drug of choice in angular conjunctivitis is

a. Oxytetracycline,

b. Chlormycetin,
44
Ophthalmology MCQs ……………………………………………………….…. Tanta university

c. Ciprofloxacin,

d. Gentamycin

Type of discharge in spring catarrh is

a. Ropy white discharge,

b. Purulent discharge,

c. Mucopurulent discharge,

d. Orange fluorescent discharge.

Which of the following is contraindicated in the treatment of acute congestive glaucoma

a.atropine

b.pilocarpine

c.timolol

d.dorzolamide

Second sight is seen in


a. Lental sclerosis,

b. Morgagnian,

c. Nuclear cataract,

d. Cortical cataract.

Rosette shaped cataract is a feature of

a.concussion injury

b.diabetes mellitus

c.Congenital Rubella

d. Wilson's Disease

The Correct Answer is B

Contusion cataract is usually stellate or rosette-shaped involving the axial posterior lens capsule

4) After cataract refers to :

a) Refractive error following cataract surgery

b) Opacity following ECCE B

c) A complication following ICCE

d) Intraocular implants after cataract extraction

45
Ophthalmology MCQs ……………………………………………………….…. Tanta university

5) Seasonal allergic conjunctivitis is :

a) Bilateral & recurrent

b) Associated with watery colorless secretion A

c) Persists throughout life

d) Topical steroids are the mainstay of therapy

6) Which of the following is a cause of unilateral amblyopia?

a) Uremia

b) Meningitis

c) Hysteria D

d) High anisometropia

7) A vitreous aspirate has been collected in an emergency at 9 pm. What advice you like

to give to the staff on duty regarding the overnight storage of the sample:

a) The sample should be kept at 4 Degrees C

b) The sample should be incubated at 37 Degrees C B

c) The sample should be refrigerated in deep freezer. .

d) The sample should be refrigerated for the initial 3 hours and then incubated at 37

Degrees C

8) A 20 yr old man complains of difficulty in reading the news paper with his right eye,

3 wks after sustaining a gunshot injury to his left eye. The most likely diagnosis is -

a) Macular edema

b) Sympathetic ophthalmia B
c) Optic nerve avulsion

d) Delayed Vitreous hemorrhage

9) A recurrent bilateral conjunctivitis occuring with the onset of hot weather in young

boys with symptoms of burning, itching, and lacrimation with raised polygonal areas in

the palpebral conjunctiva is:

a) Trachoma

b) Phlyctenular conjunctivitis

c) Mucopurulent conjunctivitis D

d) Vernal keratoconjunctivitis

10) A patient is on follow up with you after enucleation of a painful blind eye. After

enucleation of the eyeball, a proper sized artificial prosthetic eye is advised after a

postoperative period of:

46
Ophthalmology MCQs ……………………………………………………….…. Tanta university

a) about 10 days

b) About 20 days D

c) 6 - 8 wks

d) 12 - 24 wks

11) A patient using contact lens develops corneal infection. Laboratory diagnosis of

acanthamoeba keratitis was established. The following is the best drug for treatment:

a) Propamidine

b) Neosporine

c) Ketoconazole A

d) Polyhexamethylene biguanide

12) Hereditary retinoblastomas develop the following chromosomal deletion:

a) 13q14 .

b) 13p14 A

c) 14p13

d) 14q13

13) A 1 yr old child having leucocoria was detected to be having a unilateral, large

retinoblastoma filling half the globe. Current therapy would involve:

a) Enucleation

b) Chemotherapy with local dyes

c) Direct Laser Ablation using photodynamic cryotherapy A

d) Scleral radiotherapy followed by chemotherapy


14) Type IV hypersensitivity to Mycobacterium tuberculosis antigen may manifest as:

a) Iridocyclitis

b) Polyarteritis nodosa C

c) Phlyctenular conjunctivitis

d) Giant cell arteritis

15) In a patient with AIDS chorioretinitis is typically caused by:

a) Cytomegalovirus

b) Toxoplasma gondii

c) Cryptococcus neoformans A

d) Histoplasma capsulatum

47
Ophthalmology MCQs ……………………………………………………….…. Tanta university

16) The operation of plication of inferior lid retractors is indicated in:

a) Senile ectropion

b) Senile entropion

c) Cicatricial entropion B

d) Paralytic entropion

17) Fasanella Servat operation is specifically indicated in:

a) Congential ptosis

b) Steroid induced ptosis D

c) Myasthenia gravis

d) Horner's syndrome

18) A lady wants LASIK surgery for her daughter. She asks for your opinion. All of the

following things are suitable for performing LASIK except:

a) Myopia of -4D

b) Age of 15 years

c) Stable refraction for 1 yr B

d) Corneal thickness of 600 microns

19) A child has got a congenital cataract involving the visual axis which was detected by

the parents right at birth. This child should be operated:

a) Immediately

b) At 2 months of age
c) At 1 yr of age when the globe becomes normal sized A

d) After 4 yrs of age when the entire ocular and orbital growth become normal

20) Vortex vein invasion is commonly seen in:

a) Retinoblastoma

b) Malignant melanoma

c) Optic nerve glioma B

d) Medullo-epitheliomas

1. The corneal epithelium is :

a- Keratinized stratified squamous epithelium.

b- Non keratinized stratified squamous epithelium. B

c- Tall columnar epithelium.

d- Cuboidal epithelium.

48
Ophthalmology MCQs ……………………………………………………….…. Tanta university

2. One of these factors contributes to corneal transparency:

a- Regular arrangement of stromal collagen fibrils.

b- Intact epithelium and endothelium. A

c- Normal I.O.P.

d- All of above.

3. Endothelial cell layer of the cornea are examined by :

a- Ophthalmoscope.

b- Gonioscope.

c- Specular microscope. C

d- Skiascopy.

4. Corneal diameter is measured by:

a- Slit lamp.

b- Caliper & Ruler. B

c- Keratometry.

d- Retinoscopy.

5. Corneal thickness is measured by:

a- Pachymetry.

b- Biometry.

c- Keratometry. A

d- Perimetry.

6. Corneal power and curvature is measured by:


a- Topography.

b- Keratometry.

c- Slit lamp biomicroscopy. D

d- Both A & B.

7. Only one organism of the following can invade normal corneal epithelium :

a- Psuedomonas.

b- Gonococcus.

c- Staphylococcus. B

d- Pneumococcus.

8. Healing of corneal ulcer results into an opacity because:

a- New fibres are not regularly arranged.

b- B.M is not regenerated. D

c- None of above.

d- All of above.

9. The type of corneal opacity that affects vision more is :

a- Diffuse nebula.

b- Diffuse macula.

c- Dense leucoma.

d- None of them.

10. Pseudo - cornea is formed of:

a- All corneal layers.

b- Three layers namely epithelium, stroma & endothelium. B

c- Stromal layer with epithelium.


d- Only epithelial layer.

49
Ophthalmology MCQs ……………………………………………………….…. Tanta university

11. The followings are true about hypopyon except:

a- It is leucocytosis due to bacterial toxins.

b- It is fluid & cells.

c- It is absorbed with therapy. D

d- It is infected fluid containing pus cells.

12. Ulcer serpens is caused by :

a- Staphylococci.

b- Streptococci. C

c- Pneumocucci.

d- Gonococci.

13. Steroids are indicated topically in :

a- Hypopyon ulcer.

b- Dendritic ulcer. D

c- Mycotic ulcer.

d- Disciform keratitis.

14. Mooren's ulcer is :

a- Degenerative ulcer.

b- Infective ulcer. C

c- Auto immune ulcer.

d- Neuroparalytic ulcer.
15. The reservoir of infection in herpes zoster ophthalmicus is:

a- Ciliary ganglion. '

b- Gasserian ganglion;

c- Superior cervical ganglion.

d- Inferior cervical ganglion.

16. Fleischer's ring on the corneal epithelium is seen in :

a- Keratoglobas.

b- Keratoconus.

c- Keratomalacia

d- Anterior staphyloma.

17. In advanced keratoconus, the best treatment is :

a- Penetrating keratoplasty.

b- Soft Contact lenses.

c- Hard contact lenses. A

d- Refractive surgery.

18. In recurrent neuroparalytic keratitis the best treatment is :

a- Antibiotic drops & ointment..

b- Artificial tears.

c- Tarsorrhaphy. C

d- Closure of lacrimal puncta.

19. Small peripheral corneal perforation leads to:

a- Corneal fistula.
b- Anterior polar cataract.

c- Peripheral anterior synechia. C

d- Anterior staphyloma.

20. A patient with corneal ulcer noticed sudden cessation of pain & relieve of other symptoms. The
possible

occurrence is:

a- Complete cure.

b- Perforation.

51
Ophthalmology MCQs ……………………………………………………….…. Tanta university

c- Endophthalmitis. A

d- Corneal fistula.

1. The corneal epithelium is :

a- Keratinized stratified squamous epithelium.

b- Non keratinized stratified squamous epithelium.

c- Tall columnar epithelium.

d- Cuboidal epithelium.

2. One of these factors contributes to corneal transparency:

a- Regular arrangement of stromal collagen fibrils.

b- Intact epithelium and endothelium.

c- Normal I.O.P.

d- All of above.

3. Endothelial cell layer of the cornea are examined by :

a- Ophthalmoscope.

b- Gonioscope.

c- Specular microscope.

d- Skiascopy.

4. Corneal diameter is measured by:

a- Slit lamp.

b- Caliper & Ruler.

c- Keratometry.
d- Retinoscopy.

5. Corneal thickness is measured by:

a- Pachymetry.

b- Biometry.

c- Keratometry.

d- Perimetry.

6. Corneal power and curvature is measured by:

a- Topography.

b- Keratometry.

c- Slit lamp biomicroscopy.

d- Both A & B.

7. Only one organism of the following can invade normal corneal epithelium :

a- Psuedomonas.

b- Gonococcus.

c- Staphylococcus.

d- Pneumococcus.

8. Healing of corneal ulcer results into an opacity because:

a- New fibres are not regularly arranged.

b- B.M is not regenerated.

c- None of above.

d- All of above.

9. The type of corneal opacity that affects vision more is :


a- Diffuse nebula.

b- Diffuse macula.

c- Dense leucoma.

d- None of them.

10. Pseudo - cornea is formed of:

a- All corneal layers.

b- Three layers namely epithelium, stroma & endothelium.

51
Ophthalmology MCQs ……………………………………………………….…. Tanta university

c- Stromal layer with epithelium.

d- Only epithelial layer.

11. The followings are true about hypopyon except:

a- It is leucocytosis due to bacterial toxins.

b- It is fluid & cells.

c- It is absorbed with therapy.

d- It is infected fluid containing pus cells.

12. Ulcer serpens is caused by :

a- Staphylococci.

b- Streptococci.

c- Pneumocucci.

d- Gonococci.

13. Steroids are indicated topically in :

a- Hypopyon ulcer.

b- Dendritic ulcer.

c- Mycotic ulcer.

d- Disciform keratitis.
14. Mooren's ulcer is :

a- Degenerative ulcer.

b- Infective ulcer.

c- Auto immune ulcer.

d- Neuroparalytic ulcer.

15. The reservoir of infection in herpes zoster ophthalmicus is:

a- Ciliary ganglion. '

b- Gasserian ganglion;

c- Superior cervical ganglion.

d- Inferior cervical ganglion.

16. Fleischer's ring on the corneal epithelium is seen in :

a- Keratoglobas.

b- Keratoconus.

c- Keratomalacia.

d- Anterior staphyloma.

17. In advanced keratoconus, the best treatment is :

a- Penetrating keratoplasty.

b- Soft Contact lenses.

c- Hard contact lenses.

d- Refractive surgery.

18. In recurrent neuroparalytic keratitis the best treatment is :

a- Antibiotic drops & ointment..

b- Artificial tears.
c- Tarsorrhaphy.

d- Closure of lacrimal puncta.

19. Small peripheral corneal perforation leads to:

52
Ophthalmology MCQs ……………………………………………………….…. Tanta university

a- Corneal fistula.

b- Anterior polar cataract.

c- Peripheral anterior synechia.

d- Anterior staphyloma.

20. A patient with corneal ulcer noticed sudden cessation of pain & relieve of other symptoms. The
possible

occurrence is:

a- Complete cure.

b- Perforation.

c- Endophthalmitis.

d- Corneal fistula.

retinitis pigmentosa :

a) it is usually bilateral

b)usually hereditary.

c)causes cataract

d)causes tunnel vision.

e)related to usher syndrome .

answers all are true

2) regarding retinoblastoma :

a)amaurotic cat reflex is seen .

b)divergent squint is the most common manifestation.


c)pseudorosette is specific feature .

d)pseudohypopyon is usual manifestation

e)it is usually calcified .

answers amaurotic cat reflex yellow or white pupillary reflex classically noted by parents convergent
squint is seen and the

second most common manifestation of disease .neither pseudorosette nor homer wright rosette are
specific of disease

.specificity is related with 2 F flexner rosette and fleuters.

pseudohypopyon is the rare maifestation of the disease .in 75% cases calcification is seen on x ray .

3) Central retinal vein occlusion:

a)most common cause in elderly is diabetes .

b)in younger age group facial erysiplas can do it

c)90 day galucoma can occur .

d) loss of vision is not as rapid as in retianl artery occlusion

e)typical fundual picture is thunder blood fundus.

answers all are true .

4) regarding management of retinoblastoma :

a) in stage 1 @ 2 enuclation is done

b)in stage exentration of orbit is done.

c)cobalt radiotherapy.

d)chemotherapy based on cyclophosphamide vincristine and adrimycin.

e)photocoagulation by argon laser.

answer all are true best mneumonic is ERCP ENCULEATION EXENTRATION RADIOTHERAPY
CHEMOTHERAPY
CRYOTHERAY PHOTOCOAGULATION.

5)REGARDING ELECTRORETINOGRAM

A) there are three wave pattern .

b) a wave is negative .

c) c wave is positive deflection.

d) b wave is generated from the neural layer .

e) show charactersitic pattern in retinitis pigmentosa.

53
Ophthalmology MCQs ……………………………………………………….…. Tanta university

54
MC blood transfusion reaction is:

a. Febrile non-hemolytic transfusion reaction

b. Hemolysis

c. Transmission of infections

d. Electrolyte imbalance

All of the following infections may be transmited via blood

transfusion, except:

a. Parvo B-19 b. Hepatitis G

c. Dengue virus d. Cytomegalovirus

Which of the following is the least likely complication afer

massive blood transfusion?

a. Hyperkalemia b. Citrate toxicity

c. Hypothermia d. Metabolic acidosis

4. Fresh hold blood transfusion is done with in how much time

of collection?

a. Immediately b. 1 hours

c. 4 hours d. 24 hours

5. Which of the following investigations should be done

immediately to best confrm a non matched blood transfusion

reaction? (All India 2010)

a. Indirect Coomb’s test

b. Direct Coomb’s test

c. Antibody in patient’s serum

d. Antibody in donor serum

6. Blood components products are: (PGI Dec 2005)

a. Whole blood b. Platelets

c. Fresh frozen plasma d. Leukocyte reduced RBC


e. All of the above

7. A man is rushed to casualty, nearly dying afer a massive

blood loss in an accident. There is not much time to match

blood groups, so the physician decides to order for one of the

following blood groups. Which one of the following blood

groups should the physician decide: (AIIMS June 2004)

a. O negative b. O positive

c. AB positive d. AB negative

8. One unit of fresh blood arises the Hb% concentration by:

a. 0.1 gm% b. 1 gm% (All India 2003)

c. 2 gm% d. 2.2 gm%

9. Which of the following statements about acute hemolytic

blood transfusion reaction is true? (PGI June 2004)

a. Complement mediated hemolysis is seen

b. Type III hypersensitivity is responsible for most cases

c. Rarely life threatening

d. Renal blood flow is always maintained

e. No need for stopping transfusion

10. True about blood transfusions: (PGI June 98)

a. Antigen ‘D” determines Rh positivity

b. Febrile reaction is due to HLA antigens

c. Anti-d is naturally occurring antibody

d. Cryoprecipitate contains all coagulation factors

11. Which of the following is beter indicator of need for

transfusion? (AIIMS 80, UPSC 87)

a. Urine output b. Hematocrit

c. Colour of skin d. Clinical examination


12. Massive blood transfusion is defned as: (PGI 95)

a. 350 ml in 5 min b. 500 ml in 5 min

c. 1 litre in 5 min d. Whole blood volume

13. How long can blood stored with CPDA? (JIPMER 2003)

a. 12 days b. 21 days

c. 28 days d. 48 days

14. Massive transfusion in previous healthy adult male can

cause hemorrhage due to: (PGI 98)

a. Increased t-PA

b. Dilutional thrombocytopenia

c. Vitamin K defciency

d. Decreased fbrinogen

15. Arterial blood gas analysis in a botle containing heparin

causes a decrease in value of:

a. pCO2 b. HCO3

c. pH d. All of the above

16. Massive blood transfusion is defned as:

a. Whole blood volume in 24 hours (Recent Questions 2013)

b. Half blood volume in 24 hours

c. 40% blood volume in 24 hours

d. 60% blood volume in 24 hours

BLOOD TRANSFUSION COMPLICATIONS

17. Afer blood transfusion the febrile non-hemolytic transfusion reaction (FNHTR)
occurs due to?

a. Alloimmunization

b. Antibodies against donor leukocytes and HLA Ag

c. Allergic reaction

d. Anaphylaxis
18. Blood grouping and cross-matching is must prior to infusion

of: (MHPGMCET 2007)

a. Gelatin b. Dextran

c. Albumin d. FFP

19. Blood grouping and cross matching is must prior to infusion

of: (MHPGMCET 2008)

a. Gelatin b. Albumin

c. Dextran d. Hemaceal

20. Collection of blood for cross matching and grouping is done

before administration of which plasma expander?

a. Hydroxyl ethyl starch b. Dextran (MHSSMCET 2007)

c. Mannitol d. Hemacele

21. Mismatched blood transfusion in anesthetic patient presents

is:

a. Hyperthermia and hypertension (PGI June 2000)

b. Hypotension and bleeding from site of wound

c. Bradycardia and hypertension

d. Tachycardia and hypertension

Eyelids, conjunctiva, cornea & sclera


1. Distichiasis is:
a. Misdirected eyelashes
b. Accessory layer of eyelashes
c. Downward drooping of upper lid
d. Outward protrusion of lower lid
2. Band shaped keratopathy is commonly caused by deposition of:
a. Magnesium salt
b. Calcium salt
c. Ferrous salt
d. Copper salt
3. Irrespective of the etiology of a corneal ulcer, the drug always
indicated is:
a. Corticosteroids
b. Cycloplegics
c. Antibiotics
d. Antifungals
4. Dense scar of cornea with incarceration of iris is known as:
a. Leucoma adherence
b. Dense leucoma
c. Ciliary staphyloma
d. Iris bombe
5. The treatment of photo-ophthalmia is:
a. Atropine
b. Steroids
c. Dark glasses
d. Patching and reassurance
6. Corneal sensations are lost in:
a. herpes simplex
b. Conjunctivitis
c. Fungal infections
d. Marginal keratitis
7. The color of fluorescein staining in corneal ulcer is:
a. Yellow
b. Blue
c. Green
d. Royal blue
8. Phylecten is due to:
a. Endogenous allergy
b. Exogenous allergy
c. Degeneration
d. None of the above
9. A recurrent bilateral conjunctivitis occurring with the onset of hot
weather in young boys with symptoms of burning, itching, and
lacrimation with large flat topped cobble stone papillae raised areas in
the palpebral conjunctiva is:
A. Trachoma
B. Phlyctenular conjunctivitis
C. Mucopurulent conjunctivitis
D. Vernal keratoconjunctivitis
10. Which of the following organism can penetrate intact corneal
epithelium?
A. Strept pyogenes
B. Staph aureus
C. Pseudomonas pyocyanaea
D. Corynebacterium diphtheriae
11. A 12 years old boy receiving long term treatment for spring catarrh,
developed defective vision in both eyes. The likely cause is;
a. Posterior subcapsular cataract
b. Retinopathy of prematurity
c. Optic neuritis
d. Vitreous hemorrhage
12. A young child suffering from fever and sore throat began to
complain of lacrimation. On examination, follicles were found in the
lower palpebral conjunctiva with tender preauricular lymph nodes. The
most probable diagnosis is:
a. Trachoma
b. Staphylococal conjunctivitis
c. Adenoviral conjunctivitis
d. Phlyctenular conjunctivitis
13. Bandage of the eye is contraindicated in:
a. Corneal abrasion
b. Bacterial corneal ulcer
c. Mucopurulent conjunctivitis
d. After glaucoma surgery
14. Ten years old boy complains of itching. On examination, there are
mucoid nodules with smooth rounded surface on the limbus, and
mucous white ropy mucopurulent conjunctivitis discharge. He most
probably suffers from:
a. Phlyctenular conjunctivitis
b. Mucopurulent conjunctivitis
c. Bulbar spring catarrh
d. Purulent conjunctivitis
15. In viral epidemic kerato-conjunctlvitis characteristically there is:
a. Copious purulent discharge
b. Copious muco-purulent discharge
c. Excessive watery lacrimation
d. Mucoid ropy white discharge
16. Corneal Herbert's rosettes are found in:
a. Mucopurulent conjunctivitis
b. Phlyctenular keratoconjunctivitis
c. Active trachoma
d. Spring catarrh
17. A patient complains of maceration of skin of the lids and
conjunctiva redness at the inner and outer canthi. Conjunctival swab is
expected to show:
a. Slaphylococcus aureus.
b. Streptococcus viridans.
c. Streptococcus pneumonae
d. Morax- Axenfield diplobacilli
18. Tranta's spots are noticed in cases of:
a. Active trachoma
b. Bulbar spring catarrh
c. Conical phlycten
d. vitamin A deficiency.
19. A painful, tender, non itchy localized redness of the conjunctiva can
be due to:
a. Bulbar spring catarrh.
b. Episcleritis.
c. Vascular pterygium.
d. Phlyctenular conjunctivitis.
20. In trachoma the patient is infectious when there is:
a. Arlt's line
b. Herbert's pits
c. Post-trachomatous concretions.
d. Follicles and papillae in the palpebral conjunctiva.
21. A female patient 18 years old, who is contact lens wearer since two
years, is complaining of redness, lacrimation and foreign body sensation
of both eyes. On examination, visual acuity was 6/6 with
negative fluorescein test. The expected diagnosis can be:
a. Acute anterior uveitis.
b. Giant papillary conjunctivitis.
c. Bacterial corneal ulcer.
d. Acute congestive glaucoma
22. Fifth nerve palsy could cause:
a. Ptosis
b. Proptosis
c. Neurotrophic keratitis
d. Lagophthalmos
23. Topical steroids are contraindicated in a case of bacterial corneal
ulcer for fear of:
a. Secondary glaucoma,
b. Cortical cataract.
c. Corneal perforation,
d. Secondary viral infection.
24. The sure diagnostic sign of corneal ulcer is
a. Ciliary injection
b. Blepharospasm
c. Miosis
d. Positive fluorescein test.
25. The effective treatment of dendretic ulcer of the cornea is:
a. Surface anesthesia
b. Local corticosteroids
c. Systemic corticosteroids
d. Acyclovir ointment,
26. Herpes simplex keratitis is characterized by:
a. Presence of pus in the anterior chamber,
b. No tendency to recurrence
c. Corneal hyposthesia.
d. Tendency to perforation.
27. Bacteria, which can attack normal corneal epithelium:
a. Neisseria gonorrhea.
b. Staphylococcal epidermidis.
c. Moraxella lacunata.
d. Staphylococcal aureus.
28. Advanced keratoconus is least to be corrected when treated by:
a. Hard contact Lens,
b. Rigid gas permeable contact Lens,
c. Spectacles.
d. Keratoplasty.
29. Trantas nodules are seen in:
a. Blepharoconjuctivitis
b. Vernal conjunctivitis
c. Phlyctenular conjunctivitis
d. Herpetic keratitis
30. Organisms causing angular conjunctivitis are:
a. Moraxella Axenfield bacilli
b. Pneumococci
c. Gonococci
d. Adenovirus
31. Chalazion is a chronic inflammatory granuloma of
a. Meibomian gland
b. Zies's gland
c. Sweat gland
d. Wolfring’s gland
32. Ophthalmia neonatorum is prevented by:
a. 1% Silver nitrate
b. Penicillin drops
c. Normal saline drops
d. Frequent eye washes
33. Cornea is thinned in:
a. Keratoconus
b. Fuch's dystrophy
c. Bulbous lesion
d. All of the above
34. Distichiasis is:
a. An extra row of lashes
b. Central fusion of eye brow
c. Suturing of lashes
d. Absence of lashes
e. White lashes
35. Dendritic ulcer causing virus:
a. Herpes simplex
b. Herpes zoster
c. Cytomegalovirus
d. Adenovirus
e. Poxvirus
36. Treatment of choice in angular conjunctivitis is:
a. Penicillin
b. Dexamethasone
c. Sulphacetamide
d. Zinc oxide
37. Herbert's pits are seen in:
a. Trachoma
b. Herpetic conjunctivitis
c. Ophthalmia neonatorum
d. Spring catarrh
e. Phlyctenular ulcer
38. The H.P. inclusion bodies in trachoma are:
a. Intra-nuclear
b. Intra-cytoplasmic
c. Both
d. None of the above
39. Symptoms of corneal ulcer are following except:
a. Mucopurulent discharge
b. Pain in the eye
c. Redness of the eye
d. Watering
40. Hordeolum externum is a suppurative inflammation of:
a. Zeis' gland
b. Meibomian gland
c. Moll's gland
d. Gland of Krause
e. Gland Wolfring
41. Intense itching is the most common symptom of:
a. Trachoma
b. Blephritis
c. Phlyctenular conjuctivitis
d. Spring catarrh
42. Which of the following is not a source of nutrients to cornea?
a. Air
b. Aqueous humour
c. Perilimbal capillaries
d. Vitreous humour
43. Treatment of phlyctenular conjunctivitis is:
a. Systemic Anti-TB drugs
b. Systemic steroids
c. Topical acyclovir
d. Topical steroids
44. Blood vessels in a trachomatous pannus lie:
a. Beneath the Descemet's membrane.
b. In the substantia propria.
c. Between Bowman's membrane & substantia propria.
d. Between Bowman's membrane & Epithelium.
45. Dendritic ulcer is due to:
a. Herpes simplex
b. Fungal infection
c. Syphilis
d. Phlycten
46. Not causing ophthalmia neonatorum:
a. Staphylococcus aureus
b. Adenovirus
c. TRIC
d. Gonococcus
47. The dendritic corneal ulcer is typical of:
a. Varicella-Zoster infection
b. Herpes simplex infection
c. Pseudomonas infection
d. Aspergillus infection
48. In vernal catarrh, the characteristic cells are:
a. Macrophage
b. Eosinophils
c. Neutrophils
d. Epitheloid cells
49. Cobblestone appearance of conjunctiva is seen with:
a. Trachoma
b. Spring catarrh
c. Purulent conjunctivitis
d. Angular conjunctivitis
50. In which of the following conditions in small ulcers the bleed are
seen, on removing the yellow crusts on the lid margin:
a. Squamous blepharitis
b. Ulcerative blepharitis
c. Parasitic blepharitis
d. Hordeolum internum
e. All of the above
51. In which of the following conditions, severe itching of the eye with
ropy discharge in a 1 0 years old boy with symptoms aggravating in
summer season is most likely present:
a. Trachoma
b. Vernal keratoconjunctivitis
c. Acute conjunctivitis
d. Blepharitis
52. Epidemic conjunctivitis is caused by:
a. Adenovirus
b. Herpes virus
c. EB virus
d. Papilloma virus
53. The commonest organism responsible for corneal ulcer is:
a. Staphylococci
b. Pneumococci
c. Pseudomonas
d. Candida
54. The commonest cause of hypopyon corneal ulcer is:
a. Moraxella
b. Gonococcus
c. Pneumococcus
d. Staphylococcus
e. C. diphtheria
55. Irrespective of the etiology of a corneal ulcer, the drug always
indicated is:
a. Corticosteroid
b. Atropine
c. Antibiotics
d. Antifungal
56. Vossius ring is seen in:
a. Cornea
b. Retina
c. Lens
d. Anterior chamber
57. In case of central dense leucoma 5 mm, treatment of choice is:
a. Penetrating keratoplasty
b. Lamellar keratoplasty
c. Tattooing
d. Enucleation
58. Dense scar of cornea with incarceration of iris known as:
a. Leucoma adherent
b. Leucoma non adherent
c. Ciliary staphyloma
d. Iris Bombe
e. Occlusion pupillae
59. Adhesion of iris margin anteriorly is known as:
a. Intercalary staphyloma
b. Anterior synychia
c. Posterior staphyloma
d. Iris Bombe
60. Intercalary staphyloma is a type of:
a. Equatorial staphyloma
b. Posterior staphyloma
c. Ciliary staphyloma
d. Anterior staphyloma
61. Cornea is supplied by nerve fibers derived from:
a. Trochlear nerve
b. Optic nerve
c. Trigeminal nerve
d. Oculomotor nerve
62. Ciliary Congestion is not seen in:
a. Herpetic keratitis
b. Bacterial ulcer
c. Chronic iridocyclitis
d. Catarrhal conjunctivitis
e. Acute iridocyclitis
63. Not true about herpes keratitis:
a. Steroids accelerates recovery
b. Present in trigeminal ganglia
c. Recurrent
d. Usually unilateral
64. Leucokoria is seen in:
a. Glaucoma
b. Cataract
c. Retinitis pigmentosa
d. Acute dacryocystitis
65. Most of the thickness of cornea is formed by:
a. Epithelial layer
b. Substantia propria
c. Descemet's membrane
d. Endothelium
66. Corneal ulceration is caused by injury to the —— cranial nerve
a. Third
b. Fifth
c. Sixth
d. Fourth
67. Dendritic ulcer is due to:
a. Herpes simplex
b. Fungal infection
c. Syphilis
d. Phlyctena
68. Herpetic corneal ulcer is diagnosed by:
a. Geimsa stain
b. EL1SA
c. Cell culture/PCR
d. Rose Bengal stain
69. Most common viral infection of the cornea is:
a. Herpes simplex
b. Herpes zoster
c. Adenovirus
d. Molluscum contagiosum
70. A 30 years old male presents with a history of injury to the eye with
a leaf 5 days ago and pain, photophobia and redness of the eye for 2
days. What would be the most likely pathology?
a. Anterior uveitis
b. Conjunctivitis
c. Fungal corneal ulcer
d. Corneal laceration
71. Lagophthalmos occurs as a result of paralysis of:
a. 6th cranial nerve
b. 5th cranial nerve
c. Levator palpebrae-superioris
d. 7th cranial nerve
72. Hordeolum externum is inflammation of:
a. Zeis gland
b. Tarsus
c. Meibomian gland
d. Lacrimal gland
73. Chalazion is the infection of:
a. Meibomian glands
b. Conjunctiva
c. Cornea
d. Uvea
74. Ptosis and mydriasis are seen in:
a. Facial palsy
b. Peripheral neuritis
c. Oculomotor palsy
d. Sympathetic palsy
Answers
1-b 2-b 3-b 4-a 5-d 6-a
7-c 8-a 9-d 10-d 11-a 12-c
13-c 14-c 15-c 16-c 17-d 18-b
19-b 20-d 21-b 22-c 23-c 24-d
25-d 26-c 27-a 28-c 29-b 30-a
31-a 32-a 33-a 34-a 35-a 36-d
37-a 38-b 39-a 40-a 41-d 42-d
43-d 44-d 45-a 46-b 47-b 48-b
49-b 50-b 51-b 52-a 53-b 54-c
55-b 56-c 57-a 58-a 59-b 60-c
61-c 62-d 63-a 64-b 65-b 66-b
67-a 68-d 69-a 70-c 71-d 72-a
73-a 74-c

Lacrimal System
1. In DCR, the opening is made at:
a. Superior meatus
b. Middle meatus
c. Inferior meatus
2. Schirmer’s test is used for diagnosing:
a. Dry eye
b. Infective keratitis
c. Watering eyes
d. Used in Horner’s syndrome
3. All are seen in Horner’s syndrome except:
a. Miosis
b. Ptosis
c. Anhydrosis
d. Exophthalmos
4. 3 months old infant with watering lacrimal sac on pressing causes
regurgitation of mucopus material. What is the appropriate treatment?
a. Dacryocystorhinostomy
b. Probing
c. Probing with syringing
d. Massage with antibiotics up to age of 6 months
e. Dacryocystectomy
Answers
1-b 2- a 3-d 4-d

Orbit
1. Most common cause of adult unilateral proptosis
a. Thyroid orbitopathy
b. Metastasis
c. Lymphoma
d. Meningioma
2. Evisceration is:
a. Excision of the entire eyeball
b. Excision of all the inner contents of the eyeball including the uveal tissue
c. Photocoagulation of the retina
d. Removal of orbit contents
3. Lagophthalmos can occur in all of the following except;
a. 7lb cranial nerve paralysis
b. 5lh cranial nerve paralysis
c. Thyrotoxic exophthalmos
d. Symblepharon
4. Ultrasonography is helpful in confirming the diagnosis of:
a. Thyroid ophthalmopathy
b. Retinitis pigmentosa
c. Subluxated clear crystalline lens
d. Central retinal vein occlusion
5. The most important symptom differentiating orbital cellulitis from
panophthalmitis is:
a. Vision
b. Pain
c. Redness
d. Swelling
6. Axial proptosis can be due to either of these except:
a. Optic nerve glioma.
b. Lacrimal gland tumour.
c. Orbital cellulitis,
d. Thyrotoxicosis.
7. A patient complains of severe ocular pain in the right eye and
headache. The patient is drowsy, feverish with right proptosis, lid and
conjunctival edema, mastoid edema and bilateral papilledema. The
diagnosis is:
a. Endophthalmitis
b. Panophthalmitis
c. Orbital cellulitis
d. Cavernous sinus thrombosis
8. The commonest cause of bilateral proptosis is:
a. Orbital cellulitis.
b. Dysthyroid ophthalmopathy.
c. Orbital emphysema.
d. Cavernous sinus thrombosis,
9. The most common cause of unilateral proptosis in adult is;
a. lacrimal gland tumours
b. orbital cellulitis
c. panophthalmitis
d. thyroid diseases
10. Proptosis is present in the following condition except:
a. Homer's syndrome
b. Orbital cellulitis
c. Thyroid ophthalmopathy
d. Cavernous sinus thrombosis
11. The commonest cause of bilateral proptosis is:
a. Orbital cellulitis
b. Dysthyroid ophthalmology
c. Orbital emphysema
d. Cavernous sinus thrombosis
12. The commonest cause of unilateral exophthalmos is:
a. Thyroid eye disease
b. Lacrimal gland tumour
c. Orbital cellulitis
d. Cavernous sinus thrombosis
Answers
1-a 2-b 3-b 4-a 5-a 6-b
7-d 8-b 9-d 10-a 11-b 12-a

Uvea
1. All of the following are part of uvea except:
a. Pars plicata
b. Pars plana
c. Choroid
d. Schwalbe’s line
2. One of the earliest features of anterior uveitis includes:
a. Keratic precipitates
b. Hypopyon
c. Posterior synechiae
d. Aqueous flare
3. In anterior uveitis the pupil is generally:
a. of normal size
b. Constricted
c. Dilated
4. Koeppe’s nodules are found in:
a. Cornea
b. Sclera
c. Iris
d. Conjunctiva
5. Aqueous humour is formed by:
a. Epithelium of ciliary body
b. Posterior surface of iris
c. Lens
d. Pars plana
6. Secondary glaucoma due to acute attack of iridocyclitis can be
managed by the following except;
a. Corticosteroids
b. Beta-blockers
c. Carbonic anhydrase inhibitors
d. Miotics
7. Aqueous humor is secreted by:
a. Circulus iridis minor
b. Ciliary muscle
c. Ciliary processes
d. Iris crypts
8. The differential diagnosis of acute iridocyclitis includes the following
conditions except:
a. Corneal ulcer
b. Open angle glaucoma
c. Acute conjunctivitis
d. Angle closure glaucoma
9. A middle aged female with recurrent joint pain gave past history of
recurrent attacks of pain, redness and dropped vision in one eye.
Recently she complained of similar attack. The most important
diagnostic sign of activity is:
a. Festooned pupil
b. Aqueous flare and cells
c. Pigmented KPs
d. Patches of iris atrophy
10. Marcus Gunn Pupil indicates disease of:
a. Lens
b. Sclera
c. Retina
d. Optic nerve
11. The most diagnostic sign of anterior uveitis is:
a. Aqueous flare
b. Keratic precipitates
c. Constriction of pupil
d. Raised intra-ocular tension
12. Amaurotic cat's eye reflex is seen in:
a. Papilloedema
b. Retinoblastoma
c. Papillitis
d. Retinitis
13. Of the following which is the shortest acting mydriatic:
a. Tropicamide
b. Homatropine
c. Cyclopentolate
d. Atropine
14. ‘D’ shaped pupil is seen in:
a. Iridocyclitis
b. Iridodialysis
c. Glaucoma
d. Dislocation of lens
15. Uveitis is characterized by all except:
a. Mucopurulent discharge
b. Small pupils
c. Moderate pain
d. Marked tenderness
16. Small pupils are seen in:
a. Argyll Robertson pupil
b. Adie pupil
c. IIIrd nerve palsy
d. Mydriatic drops instillation
17. During accommodation all the following changes occur except:
a. Pupil contracts
b. Anterior chamber shallows
c. Lens becomes thinner
d. Anterior and posterior surface of the lens become convex
e. Lens sinks down
18. Features of iritis include all of the following, except:
a. Small pupil
b. Normal pupillary reaction
c. Ciliary congestion
d. Aqueous flare
e. Loss of iris pattern
19. Commonest cause of vitreous haemorrhage in a young adult is:
a. Diabetes
b. Trauma
c. Periphlebitis reuinac
d. Central retinal vein occlusion
20. All of the following statements about the treatment of acute anterior
uveitis are true except:
a. Mydriasis is important
b. Steroids should be avoided
c. It is usually not necessary to admit the patient to the hospital
d. Investigations for systemic disease are often negative
21. The earliest feature of anterior uveitis includes:
a. Keratic precipitates
b. Hypopyon
c. Posterior synechiae
d. Aqueous flare
22. Dilated pupil is seen in all of the following except:
a. Pontine hemorrhage
b. Optic trophy
c. Acute glaucoma
d. Papillitis
23. Drug of choice for Acute Iridocylitis is
a. Acetazolamide
b. Atropine
c. Antibiotics
d. Aspirin
24. Iridodialysis is:
a. Iris is broken from ciliary body
b. Hole in iris
c. Tremulousness of iris
d. All of the above
25. Atropine sulphate 1% drop or ointment must be used to apically in:
a. Acute conjunctivitis
b. Chronic conjunctivitis
c. Iritis
d. Acute congestive glaucoma
26. Commonest cause of posterior staphyloma is:
a. Glaucoma
b. Retinal detachment
c. Iridocyclitis
d. High myopia
27. The following is a part of uvea except:
a. Bowman's membrane
b. Iris
c. Ciliary body
d. Choroid
Answers
1-d 2-d 3-b 4-c 5-a 6- d
7-c 8-b 9-b 10-d 11-b 12-b
13-a 14-b 15-a 16-a 17-c 18-b
19-b 20-b 21-d 22-a 23-b 24-a
25-c 26-d 27-a

Lens
1. Which laser is used for capsulotomy?
a. Diode laser
b. Carbon dioxide laser
c. Excimer laser
d. Yag laser
2. Unilateral aphakia is likely to be corrected by any of the following
except:
a. Anterior chamber intraocular lens
b. Posterior chamber intraocular lens
c. Contact tens
d. Glasses
3. Phakolytic glaucoma is best treated by:
a. Fistulizing operation
b. Cataract extraction
c. Cyclo-destructive procedure
d. Miotics and Beta blockers
4. Lens induced glaucoma least occur in:
a. Intumescent cataract.
b. Anterior lens dislocation,
c. Posterior subcapsular cataract
d. Posterior lens dislocation
5. After 48 hours of a cataract extraction operation, a patient
complained of ocular pain and visual loss. On examination, this eye
looked red with ciliary injection, corneal oedema and absent red reflex.
The first suspicion must be:
a. Secondary glaucoma.
b. Anterior uveitis.
c. Bacterial endophthalmitis.
d. Acute conjunctivitis
6. Treatment of choice in Aphakia is:
a. Spectacles
b. Contact lens
c. Anterior chamber 1OL
d. Posterior chamber 1OL
7. The etiology of complicated cataract all, except:
a. Disciform keratitis
b. Iridocyclitis
c. Retinitis pigmentosa
d. Retinal detachment
8. Cylindrical lenses are used in:
a. Astigmatism
b. Myopia
c. Hypermetropia
d. Presbyopia
9. Exact site where intraocular lens is commonly fitted:
a. Capsular ligament
b. Endosulcus
c. Ciliary supported
D. Capsular bag
10. Earliest visual rehabilitation occurs with:
a. Phacoemulsification
b. 1CCE+1OL
c. ECCE+1OL
d. 1CCE alone or laser
Answers
1- d 2- d 3-b 4-c 5-c
6-d 7-a 8-a 9-d 10-a

Glaucoma
1. Pneumato-tonometer is a type of:
a. Goniolens
b. Tonometer
c. Perimeter
d. Tonography
2. When there is intumescene of the lens causing glaucoma?
a. Trephining should be done
b. Paracentesis is indicated
c. Lens should be extracted
d. Glaucoma should be treated medically only
3. All the following associated OAG include all the following except:
a. Roenne’s nasal step
b. Enlarged blind spot
c. General depression of isopters
d. Loss of central fields
e. Tubular vision
4. The treatment of choice for the other eye in angle closure glaucoma
is:
a. Surgical peripheral iridectomy
b. Yag laser iridotomy
c. Trabeculotomy
d. Trabeculectomy
5. Topical atropine is contraindicated in:
a. Retinoscopy in children
b. Iridocyclitis
c. Corneal ulcer
d. Primary angle closure glaucoma
6. The following type of glaucoma is improved with mydriatics:
a. Primary open angle glaucoma.
b. Infantile glaucoma.
c. Phacolytic glaucoma.
d. Pupillary block glaucoma.
7. A newly born, presented with big hazy cornea and photophobia.
His management begins with:
a. Electrophysiological study of the retina,
b. Syringing of naso-lacrimal duct.
c. Fluorescein stain to the cornea.
d. Measuring his ocular tension.
8. Total unilateral blindness (no PL) is a feature of:
a. Corneal ulcer
b. Mature senile cataract
c. Absolute glaucoma
d. Macular degeneration
9. When visual deterioration in a glaucomatous patient reaches no PL
the diagnosis is:
a. Acute congestive glaucoma
b. Chronic congestive glaucoma
c. Chrome open angle glaucoma
d. Absolute glaucoma
10. In acute angle closure glaucoma the pupil is:
a. Rounded, irreactive and dilated
b. Pin point constricted
c. Oval, vertically dilated
d. Normal sized, reactive
11. Which of the following conditions need immediate hospitalization?
a. Central retinal vein occlusion
b. Open angle glaucoma
c. Denderitic ulcer
d. Acute angle closure glaucoma
12. Glaucoma field changes included all the following except:
a. Arcuate scotoma.
b. Hemianopia
c. Enlarged blind spot
d. Centrocecal scotoma.
13. In case of buphthalmos we usually find:
a. shallow anterior chamber
b. Large Hazy cornea > 13mm.
c. Leukocoria
d. Occlusio pupille
14. 100 days glaucoma is seen in:
a. Central retinal artery occlusion
b. Central retinal vein occlusion
c. Neovascular glaucoma
d. Steroid induced glaucoma
15. Neovascular glaucoma follows:
a. Thrombosis of central retinal vein
b. Acute congestive glaucoma
c. Staphlococcal infection
d. Diabetes mellitus
e. Inference of aqueous drainage by Schlemm's canal mechanism.
16. Early features of chronic simple glaucoma include all except:
a. Mild headache
b. Acute onset
c. Frequent change of persbyopic glasses
d. Blurred vision
e. Clear cornea
17. When there is intumescence of the lens causing glaucoma:
a. Trephining should be done
b. Paracentesis is indicated
c. Lens should be extracted
d. Glaucoma should be treated medically
18. Increased ocular pressure in Buphthalmos causes all of the following
except:
a. Stretching of sclera
b. Corneal vascularization
c. Corneal curvature prominence
d. Rupture of Descemet's membrane
19. Treatment of choice for fellow eye in angle closure glaucoma is:
a. Timolol
b. Pilocarpine
c. Yag Laser iridectomy
d. Surgical iridectomy
20. In chronic simple glaucoma, the most common earliest field defect
is:
a. Arcuate field defect
b. Baring of blind spot
c. Bjerrum scotoma
d. Siedel's sign
21. In early glaucomatous cupping, disc is:
a. Round
b. Oval vertically
c. Oval horizontally
d. Pinpoint
22. In a patient with acute glaucoma the prophylactic treatment for
other eye is:
a. Peripheral iridectomy
b. Schele's operation
c. Anterior sclerotomy
d. Iridenclesisis
23. All the following anatomical changes will predispose to primary
angle closure glaucoma except:
a. Small cornea
b. Flat cornea
c. Anterior chamber shallow
d. Short axial length of eye ball
24. You have been referred a case of open angle glaucoma. Which of the
following would be an important point in diagnosing the case?
a. Shallow anterior chamber
b. Optic disc cupping
c. Narrow angle
d. visual acuity and refractive error
25. Which one of the following drugs is contraindicated in the treatment
of primary glaucoma?
a. Pilocarpine
b. Homatropine
c. Acetazolamide
d. Timolol
Answers
1- b 2- c 3- d 4- b 5-d
6-d 7-d 8-c 9-d 10-c
11-d 12-b 13-b 14-b 15-a
16-b 17-c 18-b 19-c 20-b
21-b 22-a 23-b 24-b 25-b

Retina
1. Number of layers in neurosensory retina is:
a. 9
b. 10
c. 11
d. 12
2. In retinal detachment, fluid accumulates between:
a. Layers of outer plexiform layer and inner nuclear layer.
b. Neurosensory retina and layer of retinal pigment epithelium
c. Nerve fiber layer and rest of retina.
d. Between the retinal pigment epithelium and Bruch’s membrane.
3. 100 days glaucoma is seen in:
a. CRAO
b. BRAO
c. CRVO
d. BRVO
4. A young patient with sudden painless loss of vision, with systolic
murmur and ocular examination reveals a cherry red spot with clear
AC, the likely diagnosis is:
a. CRAO
b. CRVO
c. DM
d. BRVO
5. Triad of retinitis pigmentosa includes:
a. Waxy disc pallor
b. Arteriolar attenuation
c. Macular hypopigmentation
d. Bone specule pigmentation
6. Retinitis pigmentosa most commonly presents as:
a. Night blindness
b. Diplopia
c. Scotoma
d. Bitemporal hemianopia
7. In CRAO, a cherry red spot is due to:
a. Hemorrhage at macula
b. Increased choroidal perfusion
c. Increase in retinal perfusion at macula
d. The contrast between pale retina and reddish choroids
8. The most common primarily intraocular malignancy in adults is:
a. Retinoblastoma
b. Choroidal melanoma
c. Squamous cell carcinoma of conjunctiva
d. Iris nevus
9. Which of the following is ideal for evaluating macular functions in a
patient whose vision is only hand movement (HM)?
a. Color perception
b, Confrontation test
c. Light projection
d, Light perception
10. Night blindness is caused by:
a. Central retinal vein occlusion
b. Dystrophies of retinal rods
c. Dystrophies of the retinal cones
d. Retinal detachment
11. A patient of old standing diabetes mellitus noticed sudden muscae
volitanes. On examination, the red reflex was dim, with no details of
fundus could be seen. He might have:
a. Non proliferative diabetic retinopathy
b. Cystoid macular edema
c. Vitreous hemorrhage
d. Central retinal vein occlusion
12. Occlusion of the lower nasal branch of the central retinal artery
results in one of the following filed defects:
a. Lower nasal sector filed defect
b. Upper nasal sector filed defect
c. Upper temporal filed defect
d. Lower temporal sector filed defect
13. Which of the following is true of Diabetic retinopathy?
a. Always associated with hypertension
b. Seen only in uncontrolled diabetes
c. Incidence increases with duration of disease
d. Determines prognosis of the disease
14. Enlargement of blind spot is a sign of:
a. Avulsion of Optic nerve
b. Papillitis
c. Papilledema
d. Retinal detachment
15. Commotio retinae is seen in:
a. Concussion injury
b. Papilloedema
c. Central vein thrombosis
d. Central artery thrombosis
16. Simple retinal detachment is usually due to:
a. Tumour
b. Choroidal haemorrhage
c. Exudative retinitis
d. None of the above
17. Cherry red macula is seen in:
a. Acute Congestive glaucoma
b. Uveitis
c. Central retinal artery occlusion
d. Central retinal vein occlusion
18. Which is not found in papilloedema?
a. Blurred vision
b. Blurred margins of disc
c. Cupping of disc
d. None
19. Retinal change specific in proliferative diabetic retinopathy is:
a. Microaneurysm
b. Soft cotton wool exudates
c. A-V shunt
d. Neovascularization
20. Commonest lesion which hinders vision in diabetic retinopathy is:
a. Macular oedema
b. Microaneurysm
c. Retinal hemorrhage
d. Retinal detachment
21. Which of the following not found in diabetic retinopathy on fundus
examination:
a. Microaneurysms
b. Retinal hemorrhages
c. Arteriolar dilatation
d. Neovascularisation
Answers
1-a 2-b 3-c 4-a
5-c 6-a 7-d 8-b
9- a 10-b 11-c 12-c
13-c 14-c 15-a 16-d
17-c 18-c 19-d 20-a
21-c

Optic Nerve
1. Primary optic atrophy results from:
a. Retinal disease
b. Chronic glaucoma
c. Papilledema
d. Neurological disease
2. Retro-bulbar optic neuritis is characterized by:
a. Marked swelling of the optic disc.
b. Impaired direct light reflex in the affected eye
c. Impaired consensual light reflex in the affected eye
d. Normal visual acuity
3. The type of optic atrophy that follows retro-bulbar neuritis is:
a. Secondary optic atrophy
b. Connective optic atrophy
c. Glaucomatous optic atrophy
d. Primary optic atrophy
4. A male patient 30 years old with visual acuity of 6/6 in both eyes.
Twelve hours ago he presented with drop of vision of the left eye. On
examination, visual acuity was 6/6 in the right eye and 6/60 in the left
eye. Fundus examination showed blurred edges of the left optic disc.
The most probable diagnosis is:
a. Raised intra cranial pressure
b. Raised ocular tension
c. Central retinal artery occlusion
d. Optic neuritis
5. The type of optic atrophy that following retrobulbar neuritis is:
a. Secondary optic atrophy
b. Consecutive optic atrophy
c. Glaucomatous optic atrophy
d. Primary optic atrophy
6. Injury to optic tract produces:
a. Homonymous hemianopia
b. Bitemporal hemianopia
c. Binasal hemianopia
d. Sparing of macular vision
7. All of the following statements regarding the oculomotor nerve are
true except:
a. It accommodates the eve
b. It raises the upper eyelid
c. It innervates lateral rectus
d. It constricts the pupil
8. Blurring of disc margin is found in:
a. Papillitis
b. Retrobulbar neuritis
c. Tobacco amblyopia
c. all of the above
9. In optic neuritis the best investigation to be done include:
a. Gold perimetry
b. Keratoscopy
c. Ophthalmoscopy
d. Ophthalmodynamometry
10. Elevation of the disc margin is seen in:
a. Optic atrophy
b. Papillitis
c. Papilloedema
d. None of the above
11. Optic disc diameter is:
a. 1 mm
b. 1.5 mm
c. 2 mm
d. 3 mm
12. Optic nerve function is best studied by:
a. Ophthalmoscope
b. Retinoscope
c. Perimetry
d. Goniometry
13. In optic neuritis, following are seen except:
a. Headache and vomiting
b. Pain on movement of eye
c. Sudden loss of vision
d. Afferent of pupillary reflex lost
14. A 25 year old lady suddenly develops unilateral loss of vision
decreasing from 6/6 to 4/60 with ill-sustained reaction of the pupil. She
complains of slight headache and some pain in the orbit when looking
upward. The most likely diagnosis is:
a. Acute frontal sinusitis
b. Acute iritis
c. Brain tumour
d. Retrobulbar neuritis
e. Acute congestive glaucoma
15. The test that would aid most in the diagnosis of the previous case is:
a. X-ray of the orbit
b. Visual field determination
c. Tonometry
d. Exophthalmometry
e. Gonioscopy
16. A boy presents two weeks after an injury to his left eye. He
complains of bilateral pain and watering from right. What is the
probable diagnosis?
a. Endophthalmitis
b. Optic neuritis
c. Sympathetic irritation
d. Sympathetic ophthalmitis
17. All are seen in 3rd nerve palsy except:
a. Ptosis
b. Diplopia
c. Miosis
d. Outwards eye deviation
18. Homonymous hemianopia is due to lesion at:
a. Optic tract
b. Optic nerve
c. Optic chiasma
d. Retina
e. Occipital cortex
Answers
1- d 2- b 3- a 4- d 5- d
6-a 7-c 8-a 9-a 10-c
11-b 12-c 13-a 14-d 15-b
16-d 17-c 18-a

Neuro-ophthalmology
1. Optic nerve axon emerges from:
a. Ganglion cells
b. Rods and cones
c. Amacrine cells
d. Inner nuclear layer
2. Papilloedema has all the following characteristic except:
a. Marked loss of vision
b. Blurring of disc margins
c. Hyperemia of disc
d. Field defect
3. Homonymous hemianopia is the result of a lesion in:
a. Optic chiasma
b. Retina
c. optic tract
d. Optic nerve
4. Mydriasis is present in all the following except:
a. Third nerve lesion
b. Pontine haemorrhage
c. Datura poisoning
d. Fourth stage of anesthesia
5. Miosis is present in all the following except:
a. Third nerve lesion
b. Horner's syndrome
c. Morphine poisoning
d. Argyl Robertson's pupil
6. The following cranial nerves are responsible for ocular motility
except:
a. Third nerve
b. Fourth nerve
c, Fifth nerve
d. Sixth nerve
7. In complete third nerve paralysis the direction of the affected eye in
the primary position is:
a. Inward
b. Outward
c. Outward and up
d. Outward and down
8. A patient 60 year old, diabetic for 20 years. He suffered acutely of
drooping of the right eyelid. On manual elevation of the lid he sees
double vision. What is your diagnosis?
a. Diabetic sixth nerve palsy
b. Oculomotor nerve palsy
c. Trochlear nerve palsy
d. Abducent nerve palsy
9. An area of reduced or absent vision within an intact visual field is
called:
a. Scotoma
b. Homonymous hemianopia
c. Heteronymous hemianopia
d. Altitudinal hemianopia
10. Which of the following pupils is dilated?
a. Pupil in Horner's syndrome
b. Adies pupil
c. Argyl Robertson pupil
d. Pupil in open angle glaucoma, with a C/D of 0.3.
11. D-shaped pupil occurs in:
a. Iridocyclitis
b. Iridodenesis
c. Cyclodialsis
d. Iridodialysis
Answers
1. a 2. a 3-c 4-b 5-a
6-c 7-d 8-b 9-a 10-b
11-d

Strabismus
1. All the following are extraocular muscle of eye except:
a. Superior rectus
b. Ciliary muscle
c. Inferior oblique
d. Superior oblique
2. The action of superior rectus is:
a. Elevation, intorsion, abduction
b. Elevation, intorsion, adduction
c. Elevation, extorsion, adduction
d. Elevation, extorsion, abduction.
3. The action of inferior oblique is:
a. Depression, extorsion, abduction
b. Depression, extorsion, adduction
c. Elevation, extorsion, adduction
d. Elevation, extorsion, abduction
4. The only extraocular muscle which does not arise from the apex of
the orbit is:
a. Superior rectus
b. Superior oblique
c. Inferior oblique
d. Inferior rectus
5. In paralytic squint:
a. Primary deviation > Secondary deviation
b. Primary deviation < Secondary deviation
c. Primary deviation = Secondary deviation
d. None of the above
6. In concomitant squint:
a. Primary deviation > Secondary deviation
b. Primary deviation < Secondary deviation
c. Primary deviation = Secondary deviation
d. None of the above
7. In paralytic squint, the difference between primary and secondary
deviation in the gaze of direction of the paralytic muscle:
a. Increases
b. Decreases
c. Remains the same
8. In grades of binocular vision; grade 2 is:
a. Simultaneous macular vision
b. Fusion
c. Stereopsis
9. The best treatment for amblyopia is:
a. Orthoptic exercises
b. occlusion
c. Surgery
d. Best treat after age 10 years
Answers
1-b 2-b 3-d 4-c
5-b 6-c 7-a 8-b
9-b
Anterior and posterior Chambers
1. Flat anterior chamber may result due to:
a. Occlusion by pupil by exudates
b. Seclusio pupillae
c. Leakage after cataract operation
d. Acute congestive glaucoma
e. Any of the above
2. Shallow anterior chamber is seen in:
a. High myopia
b. Aphakia
c. High hypermetropia
d. All of the above conditions
3. Angle of anterior chamber is studied with:
a. Indirect ophthalmoscopy
b. Gonioscopy
c. Retinopathy
d. Amblyoscope
4. Dilator pupillae is supplied by:
a. Postganglionic sympathetic from cervical plexus
b. Preganglionic sympathetic from cervical plexus
c. III Nerve
d. Postganglionic sympathetic fibers form V nerve.
Answers
1-e 2-c 3- b 4- a

Miscellaneous
1. Frequent change of presbyopic glasses is an early symptom of:
a. Closed angle glaucoma
b. Open angle glaucoma
c. Senile cataract
d. Vernal catarrh
2. Chronic use of steroids may lead to:
a. Iris atrophy
b. Glaucoma
c. Corneal opacity
d. Retinopathy
3. Sudden loss of unilateral vision and pain on movement is seen in:
a. Diabetes mellitus
b. Retrobulbar neuritis
c. Hypertension
d. IIIrd nerve palsy
4. The commonest intra-ocular tumour in children is:
a. Malignant melanoma
b. Retinoblastoma
c. Teratoid medulloepithelioma
d. Diktyoma
5. Hyphema is blood in the:
a. Ant. Chamber
b. Post. Chamber
c. Cornea
d. Conjunctiva
6. Herbert’s pits are seen on the:
a. Lid margin
b. Palpebral conjunctiva
c. Arlt's line
d. Limbus
7. Biconvex lens is used in all, except:
a. Aphakia
b. Presbyopia
c. Astigmatism
d. Hypermetropia
8. Most important complication of traumatic hyphema is:
a. Iridocyclitis
b. Iridodialysis
c. Blood staining of cornea
d.Siderosis bulbi
9. What is Tylosis?
a. Hypertrophy and dropping of eye lid
b. Inversion of eye lid
c. Senile eversion of eye lid
d. Distortion of cilia
10. Which of the following is not seen in blunt trauma to eye?
a. Hyphema
b. Iridiodialysis
c. Retinal haemorrhage
d. Double perforation
11. Fleischer ring is found in:
a. Keratoconus
b. Chalcosis
c. Argyrosis
d. Buphthalmos
e. None of the above
12. Intercalary staphyloma occurs at:
a. Area extending up to 8 mm from limbus
b. Limbus
c. Posterior pole of eye
d. Region of sclera which are perforated by vortex vein
e. Equatorial region
13. Blow out fracture of orbit commonly produces:
a. Deviation of septum
b. Retinal haemorrhage
c. Fracture of nasal bones
d. Fracture of floor of orbit
14. Following are avascular except:
a. Cornea
b. Sclera
c. Vitreous
d. Lens
15. A one-month old baby is brought with complaints of photophobia
and watering. Clinical examination shows normal tear passages and
clear but large cornea. The most likely diagnosis is:
a. Congenital dacryocystitis
b. Interstitial keratitis
c. Keratoconus
d. Buphthalmos
16. Blindness in Vit. A deficiency is due to:
a. Corneal involvement
b. Retinopathy
c. Conjunctival scarring
d. Uveitis
17. Mydriatics drugs are the following except:
a. Tropicamide
b. Cyclopentolate
c. Homatropine
d. Pilocarpine
18. Flashing of light is an earliest sign of:
a. Cataract
b. Glaucoma
c. Retinal detachment
d. Corneal opacity
19. In non-proliferation diabetes, most common cause of blindness:
a. Sub retinal Haemorrhage
b. Macular edema
c. Retinal detachment
d. Vitreous haemorrhage
20. Fixed dilated pupil is seen in:
a. Oculomotor palsy
b. Trochlear palsy
c. Abducent palsy
d. None
21. Arlt's line is seen in:
a. Vernal keratoconjunctivitis
c. Ocular pemphigoid
d. Trachoma
22. A two months old child presents with epiphora and regurgitation.
The most probable diagnosis:
a. Mucopurulent conjunctivitis
b. Buphthalmos
c. Congenital dacryocystitis
d. Encysted mucocele
23. Horner's syndrome is characterized by all of the following except:
a. Miosis
b. Enophthalmos
c. Ptosis
d. Cycloplegia
24. Posterior staphyloma is associated with:
a. Pathological myopia
b. Retinoblastoma
c. Acid injury
d. Sympathetic ophthalmia
25. Phlyctenular conjunctivitis is caused by:
a. Hypersensitivity to exogenous antigen
b. Mechanical injury
c. Hypersensitivity to endogenous antigen
d. Extended wear contact lenses
e. Mixed infections
26. Distichiasis is:
a. An extra row of lashes present behind the greyline
b. Dropping of the upper eye lid below the normal position
c. Semilunar fold of skin covering the medial canthus
d. Increased distance between the inner canthi
e. Irregular shaped defects of lid margin.
27. Which of the following is true regarding Herpes Zoster
Ophthalmicus:
a. The condition is essentially bilateral.
b. Dendritic ulcer is the main presentation.
c. Periocular skin affection is always present
d. Usually a painless condition
e. Corneal melting is a usual finding.
28. Which of the pathogens can produce corneal infections in the
presence of an intact epithelium
a. Staphylococcus
b. N. Gonorrhea
c. N. Meningitides
d. Streptococcus
e. Pneumococcus
29. In accommodative esotropia:
a. Surgery is the rule in its treatment
b. High myopia is usually present.
c. Astigmantism is usually present.
d. Due to excessive accommodation.
e. usually associated with amblyopia
30. Dense scar of cornea with incarceration of iris is known as:
a. Leucoma non adherent
b. Ciliary staphyloma
c. Iris bombe
d. Symbleparon
e. Leucoma adherent
31. Corneal sensations are lost in:
a. Conjunctivitis
b. Marginal keratitis
c. Herpetic keratitis
d. Fungal keratitis
e. Pannus siccus
32. The colour of flourescein staining in corneal ulcer is;
a. Yellow
b. Blue
c. Green
d. Royal blue
e. red
33.Arlt's line is present in:
a. Upper palpebral conjunctiva
b. Lower palpebral conjunctiva
c. Bulbar conjunctiva
d. Limbus
e. Lid margin
34. The most common complication of lagophthalmos is:
a. Purulent conjunctivitis
b. Exposure keratitis
c. Entropion
d. trichiasis
e. Chronic simple glaucoma
35. Main muscle responsible for lid elevation:
a. muller's muscle
b. Superior rectus muscle
c. Occipitofrontalis muscle
d. Superior oblique muscle
e. Levator palpebrae superioris muscle
36. Fundus changes in background diabetic retinopathy include:
a. Cotton wool spots
b. Intraretinal microvascular abnormalities (IRMAs)
c. Neovessels in the retina
d. Retinitis proliferans
e. Microaneurysms
37. Management of acute central retinal artery occlusion includes the
following except:
a. Ocular massage
b. Anterior chamber paracentesis
c. Intravenous acetozolamide
d. Miotics
e. Inhalation of carbogen
38. Treatment of choice for angular conjunctivitis is:
a. Vaso constrictors
b. Mast cell strabilizers
c. Zinc preparations
d. Large doses of Vitamin A
e. Antiviral drugs
39. Patient 60 year old, diabetic for 20 years. He suffered acutely of
dropping of the right eyelid. On manual elevation of the lid he sees
double vision. What is your diagnosis?
a. Trigeminal nerve neuropathy.
b. Trochlear nerve palsy.
c. Abducent nerve palsy.
d. Oculomotor nerve palsy.
e. Facial nerve palsy
40. Pupillary nerve fibers pass through the optic tract to:
a. The optic chiasma
b. The optic radiation.
c. The lateral genicualte body.
d. The pretectal nuclei.
e. The occipital cortex
41. Systemic hypertension can predispose to the following ocular
problem:
a. Rhegmatogenous retinal detachment
b. Muscca volitans
c. Retinal vein occlusion
d. Retinal vasculitis
e. Melanoma of the choroid
42. Pregnant lady at 27 week gestation developed marked elevation of
her blood pressure and proteinuria then developed marked diminution
of vision. It is mostly due to:
a. Central retinal vein occlusion
b. Rhegmatogenous retinal detachment
c. Central retinal artery occlusion
d. Exudative retinal detachment
e. Tractional retinal detachment
43. Consecutive optic atrophy is secondary to
a. papilloedema
b. papillitis
c. diseases of retina and choroid
d. glaucoma
e. Iridocyclitis
44. Chronic dacryocystitis increases the risk of:
a. Phlyctenular conjunctivitis
b. Vernal conjunctivitis
c. Pneumococcal corneal ulcer
d. Dendritic corneal ulcer
e. Diphtheretic conjunctivitis
45. Corneal Herbert's rosettes are found in:
a. Mucopurulent conjunctivitis
b. Phlyctenular keratoconjunctivitis
c. Active trachoma
d. Spring catarrh
e. Angular Blapharo conjunctivitis
46. In acute angle closure glaucoma the pupil is :
a. rounded, irreactive and dilated
b. Pin point constricted
c. Vertically oval, dilated, irreactive
d. Normal sized, reactive
e. D shaped pupil
47. Patient received a tennis ball hit to his eye which used to have 6/6
vision. External eye examination showed no abnormalities vision is
H.M. and red reflex is normal. Possible diagnosis is:
a. comotio retinae
b. traumatic cataract
c. vitreous hemorrhage
d. secondary glaucoma
e. Subconjunctival Heamorrhage.
48. The commonest cause of diminution of vision in a school-aged child
is:
a) Soft cataract
b) Infantile glaucoma
c) Errors of refraction
d) Corneal dystrophy
e) Retinal detachment
49. Cobble stone papillae are pathognomonic of
a. trachoma
b. Inclusion conjunctivitis
c. Vernal conjunctivitis
d. Adenoviral conjunctivitis
e. Diphtheretic conjunctivitis
50. A scan biometry is used to calculate the power of the intraocular
lens by measuring:
a. Corneal curvature
b. Refractive index
c. Depth of the anterior chamber
d. Axial length of the eye
e. Power of the cornea
51. The sure diagnostic sign of corneal ulcer
a. Ciliary injection
b. Blepharospasm
c. Miosis
d. positive fluorescein test
e. photophobia
52. Early stages of papilledema cause:
a. arcuate scotoma
b. Enlargement of the blind spot
c. ring- shaped scotoma
d. Siedle scotoma
e. Homonymous hemianopia
53. Cylindrical lenses are Prescribed for
a. Myopia
b. Hypermetropia
c. Astigmatism
d. Athenopia
e. Cataract
54. Temporal crescent is seen typically in
a. Astigmatism
b. Hypermetropia
c. Myopia
d. squint
e. Athenopia
55. Simple myopic astigmatism means
a. One meridian is myopic and the other is emmetropic
b. Both meridian are myopic
c. One meridian is hypermetropic and the other emmetropic
d. Both meridian are hypermetropic
e. Both meridian are emmetropic
56. Ptosis in Horner's syndrome, is due to paralysis of:
a. Riolan's muscle
b. Horner's muscle
c. Muller's muscle
d. The levator palpebral muscle
e. Orbicularis oculi muscle
57. Blow out fracture Most commonly affect
a. The inferior wall of the orbit
b. The medial wall of the orbit
c. The lateral wall of the orbit
d. The roof of the orbit
e. The orbital apex
58. Vossius ring is iris pigments deposited on
a. Posterior surface of the cornea
b. Anterior surface of the lens
c. Angle of the anterior chamber
d. Posterior surface of the lens
e. Anterior surface of the cornea
59. The sure sign of scleral rupture is
a. Shallow anterior chamber
b. Decreased visual acuity
c. Prolapsed intraocular contents
d. Hypotony
e. Hyphema
60. A male patient was complaining of continuous redness of both eyes,
foreign body sensation, and frequent loss of lashes. On examination, the
lid margins were hyperemic, and the lashes were matted with yellow
crusts, which left painful ulcers on trying to remove. The most reliable
diagnosis is :
a. Squamous blepharitis.
b. Cicatricial entopion.
c. Spastic entropion.
d. Ulcerative blepharitis.
e. Viral Conjunctivitis
61. Which of the following is most likely to have a disastrous visual
outcome if not treated immediately?
a. Conjunctivitis
b. Subconjunctival hemorrhage
c. Angle closure glaucoma
d. Iritis
e. Photophthalmia.
62. Ciliary congestion is most marked at the
a. Sclera
b. Fornix
c. Bulbar conjunctiva
d. Limbus
e. Iris
63. Bandage of the eye is contraindicated in:
a. Corneal abrasion
b. Bacterial corneal ulcer
c. Mucopurulent conjunctivitis
d. After glaucoma surgery
e. After cataract surgery
64. Superior oblique muscle is supplied by:-
a. Oculomtor nerve
b. Abducens nerve
c. Trochlear nerve
d. Fascial nerve
e. Trigeminal nerve
65. A patient with recent rhegmatogenous retinal detachment is
presented with:
a. Photophobia
b. Amaurosis fugax
c. Photopsia
d. Neuralgic pain
e. Red eye
66. The most characteristic in prodromal stage of acute congestive
glaucoma:
a. Coloured haloes
b. Marked ciliary injection
c. Persistent elevation of intraocular pressure
d. Profuse lacrimation
e. Profound loss of visual acuity
67. Ten years old boy complains of itching. On examination, there are
mucoid nodules with smooth rounded surface on the limbus, and
mucous white ropy discharge. He most probably suffers from:
a. Phlyctenular conjunctivitis
b. Mucopurulent conjunctivitis
c. Bulbar spring catarrh
d. Purulent conjunctivitis
e. Viral conjunctivitis
68. Which of the following is used for evaluating macular functions in a
patient whose vision is only hand movement:
a. Color perception
b. Confrontation test
c. Light projection
d. Light perception
e. Ultrasonography
69. Version movements are:
a. Binocular movements in the same direction.
b. Binocular movements in the opposite direction
c. Controlled by shirrington's low
d. Controlled by oculomotor nuclei
e. Monocular movement in a given direction
70. Cataract surgery should not be done in the presence of:
a. Corneal nebula
b. Herbert's pits
c. Arcus senilis
d. Chronic dacryocystitis
e. Arlt's line
71. Rubeosis iridis is seen in:
a. Central retinal artery occlusion
b. Central retinal vein occlusion
c. Acute iridocyclitis
d. Corneal ulcer
e. Mucopurulent conjunctivitis
72. Diminished vision in daylight is seen in:
a. Central cataract
b. Peripheral cataract
c. Peripheral corneal opacity
d. Peripheral retinal degenerations
e. Retinitis pigmentosa
73. One of the following signs disappear with medical treatment:
a. Pannus siccus
b. Tranta's spots
c. Arlt's line
d. Arcus senilis
e. Pterygium
74. Primary open angle glaucoma is not treated by:
a. Pilocarpine
b. Carbonic anhydrase inhibitors
c. Beta blockers
d. Atropine
e. Subscleral trabeculectomy
75. A 3 year old child presents with a convergent squint dating for one
year. The first step management is:
a. Proper refraction
b. Surgical correction
c. Training on the synoptophore
d. Prism prescription
e. Examination with Hess screen
76. The primary treatment of dacryocystitis in infant, should be:
a. Syringing
b. Antibiotics and massage
c. Dacryocystectomy
d. Dacryocystorhinostomy
e. Antibiotics and steroid
77. Fifth nerve palsy could cause:
a. Ptosis
b. Proptosis
c. Neurotrophic Keratitis
d. Lagophthalmos
e. Keratoconus.
78. The earliest feature of anterior uveitis includes:
a. Keratic precipitates
b. Hypopyon
c. Posterior synechiae
d. Aqueous flare
e. Peripheral anterior synechiae
79. Accommodation is maximum in
a. Adulthood
b. Childhood
c. Middle age
d. Old age
e. Women
80. Lens induced glaucoma least occur in:
a. Intumescent cataract
b. Anterior lens dislocation
c. Posterior subcapsular cataract
d. Posterior lens dislocation
e. Hypermature cataract
81. Occlusion therapy:
a. To occlude the amblyopic eye.
b. Is mandatory in alternating squint.
c. Is effective at any age
d. Is effective in apparent squint.
e. One week for each year of patient age is the maximum period for full time
occlusion
COLUMN ONE COLUMN TWO
82. Orbital excentration a. Excision of the eye ball
b. Evacuation of the content of the
83. Evisceration means
eyeball.
c. Retraction of the globe into the
orbit.
d. Non specific inflammation of the
orbital tissue.
e. Orbital periosteum and orbital
contents, lids and conjunctiva are
removed.

COLUMN ONE COLUMN TWO


84. Trichiasis a. Permanent absence of eye lashes
b. More than 4 lashes rubbing
85. Poliosis
against the cornea
86. Orbital cellulitis c. Whitening of the lashes.
d. Acute suppurative inflammation
of orbital soft tissue.
e. Lid margin is turned outwards.

COLUMN ONE COLUMN TWO


a. Chronic inflammation of lid
87. Dermatochalasis
margin
b. Acute suppurative inflammation
88. Blepharitis
of zeis gland and lash follicle
c. Incomplete closure of palpebral
89. Hordeolum internum
fissure
d. Acute suppurative inflammation
of meibomian gland.
e. Redundancy of upper eye lid skin
in old age.

COLUMN ONE COLUMN TWO


a. Gonococci
90. True membranous
b. Koch week's bacillus.
conjunctivitis is caused by
91. The commonest cause of
c. Morax Axenfield bacillus.
Mucopurulent conjunctivitis
d. Corynebacterium diphtheriae.
e. Chlamydia trachomatis.
92. IOP as measured by indentation is recorded as:
A. mm Hg
B. gm/mm2
C. a scale from 0 to 18
D. Ib/in2
93. A refractive error of +2.00 +2.00 X 180 would be classified as:
A. mixed astigmatism
B. compound hyperopic astigmatism
C. compound myopic astigmatism
D. simple hyperopic astigmatism
94. A male patient was complaining of continuous redness of both eyes, foreign body
sensation, and frequent loss of lashes. On examination, the lid margins were hyperemic,
and the lashes were matted with yellow crusts, which left painful ulcers on trying to
remove. The most reliable diagnosis is :
A. squamous blepharitis.
B. Cicatricial entopion.
C. Spastic entropion.
D. Ulcerative blepharitis.
95. Best position for intraocular lens is:
A. In the anterior chamber
B. Iris claw lens
C. In the capsular bag
D. Posterior to the capsule
96. Which of the following field defects do pituitary gland tumours produce?
A. Unilateral field defect
B. Bitemporal hemianopia
C. Homonymous hemianopia
D. Altitudinal field defect
97. A patient wearing glasses of +5.00 diopters is more prone to have:
A. Open angle glaucoma
B. Closed angle glaucoma
C. Phacolytic glaucoma
D. Neovascular glaucoma
98. The most accepted regimen in drug therapy of anterior uveitis can be:
A. Corticosteroids and antibiotics
B. Corticosteroids and miotics
C Corticosteroids and cycloplgics
D. Corticosteroids and nonsteroidal anti-inflammatory drugs.
99. A female patient 18-year-old, who is contact lens wearer since two years, is
complaining of redness, lacrimation and foreing body sensation of both eyes. On
examination, visual acuity was 6/6 with negative Fluorescein staining, the expected
diagnosis can be :
A. Acute anterior uveitis.
B. Giant papillary conjunctivitis.
C. Bacterial corneal ulcer.
D Acute congestive glaucoma.
100. Rhegmatogenous retinal detachments are more in patients with:
A. myopia
B. hyperopia
C. astigmatism
D. presbyopia
101. Slit lamp examination of your patient reveals bulging, centrally thinned corneas.
Refractometry shows an increase in astigmatism. The patient probably has:
A. flat cornea
B. keratoconjunctivitis
C. keratoconus
D. exophthalmus
102. During routine IOP mesurements, you discovered a patient with an IOP of 30-
mmHg & 25 mmHg. The C/D ratio was 0.4 and the field of vision showed paracentral
scotoma. The angle of AC was open. What is the first line of treatment?
A. Topical beta blockers
B. Laser iridotomy
C. Surgical peripheral iridectomy
D. Subscleral trabeculectomy
103. Aqueous humour is secreted by:
A. angle of anterior chamber
B. choroid
C. ciliary body
D. iris
104. Anterior chamber is shallow in
A. buphthalmos
B. open angle glaucoma
C. closed angle glaucoma
D. aphakia
105. A female patient is complaining of watering of the right eye and photophobia.
Examination showed 8 scattered rubbing lashes of the upper lid. The best treatment is :
A. Snellen's operation
B. Van Millingen's operation
C. Electroysis
D. Diathermy
106. Tremulousness of iris is seen in
A. chronic iridocyclitis
B. closed angle glaucoma
C. aphakia
D. senile immature cataract
107. White pupillary reflex is seen in
A. retinoblastoma
B. malignant melanoma
C Vitreous haemorrhage
D. Open angle glaucoma
108. The most accurate method of measuring IOP is
A. digital
B. applanation
C. air-puff non contact tonometer
D. gonioscopy
109. A patient is presenting with ocular pain, positive fluorescein staining of the cornea
with a constricted pupil, and a high intra ocular pressure. The following medication is
contraindicated:
A. Atropine
B. Pilocaprine
C. Carbonic anhydrase inhibitors
D. Beta-blockers
110. Unilateral aphakia can be treated by
A. glasses
B. intraocular lens implant
C. keratoplasty
D. radial keratotomy
111. Radial keratotomy is useful in
A. myopia
B. hypermetropia
C. presbyopia
D. aphakia
112. Optical condition of the eye in which there is a significant difference in refraction
between the two eyes
A. mixed astigmatism
B. irregular astigmatism
C. anisometropia
D. compound astigmatism
113. Chronic use of topical steroids may lead to
A. iris atrophy
B. glaucoma
C. corneal opacity
D. retinopathy
114. A young child suffering from fever and sore throat began to complain of
lacrimation. On examination, follicles were found in the lower palpebral conjunctiva
with tender preauricular lymph nodes. The most probable diagnosis is:
A. Trachoma
B. Staphylococcal conjunctivitis
C. Adenoviral conjunctivitis
D. Phlyctenular conjunctivitis
115. YAG laser is used in the treatment of
A. diabetic retinopathy
B. open angle glaucoma
C. after cataract
D. retinal detachment
116. Regarding buphthalmos, which is correct
A. shallow anterior chamber
B. bilateral
C. trabeculectomy is the treatment of choice
D. small corneal diameter (less than 10mm)
117. Treatment of choice in primary open angle glaucoma is
A. cyclodialysis
B. iridectomy
C. cyclodiathermy
D. medical
118. The optic nerve extends up to
A. optical chiasma
B. optic tract
C. lateral geniculate body
D. optic radiation
119. Consecutive optic atrophy occurs following
A. papilloedema
B. central retinal artery occlusion
C. toxic amblyopia
D. papillitis
120. 'D' shaped pupil is seen in
A. iridocyclitis
B. iridodialysis
C. glaucoma
D. dislocation of lens
121. Hordeolum externum is an inflammation of
A. lid margin
B. tarsal plate
C. meibomian gland
D. Zeis gland
122. Neovascular glaucoma can be best treated by:
A. trabeculectomy
B. pilocarpine
C. panphotocoagulation of retina
D. timolol maleate
123. Lagophthalmos is the condition of
A. incomplete closure of the palpebral aperture
B. drooping of the upper eyelid below its normal position
C. lid margin rolls outwards
D. lid margin rolls inwards
124. Paralytic ptosis is due to
A. complete or partial 3rd nerve palsy
B. 4th nerve palsy
C. 6th nerve palsy
D. 7th nerve palsy
125. The clinical features of symblepharon include
A. ectropion
B. lagophthalmos
C. chalazion
D. Xanthelasma
126. The most important examination in cases presenting with congenital ptosis is :
A. The function of the levator muscle.
B. Examination of the cornea
C. The examination of the cranial nerves.
D. The examination of the fundus.
127. Bandage of the eyes is contraindicated in:
A. Corneal abrasion.
B. Bacterial corneal ulcer.
C. Mucopurulent conjunctivitis.
D. after glaucoma surgery.
128. The first line of treatment in acid burns of the eye is
A. Patching the eye
B. Instilling a drop of oil in the eye
C. Immediate wash with plain water
D. Instilling a drop of surface anesthetic into the eye
129. An 8 year old girl received a tennis ball trauma to her right eye. On examination on
the same day you would find
A. Blood staining of the cornea
B. Hyphaema
C. Hypopyon
D. Ghost cell glaucoma
130. In myopia which one of the following is most common?
A. Equatorial staphyloma
B. Posterior staphyloma
C. Ciliary staphyloma
D. Intercalary staphyloma
131. Distichiasis is:
A. Misdirected eyelashes
B. Accessory row of eyelashes
C. Downward droping of upper lid
D. Outward protusion of lower lid
132. Arlt's line is present is:
A. upper palpebral conjunctiva
B. Lower palpebral conjunctiva
C. Bulbar conjunctiva
D. Cornea
133. Deep leucoma is best treated by:
A. Tattooing
B. Lamellar keratoplasty
C. Keratectomy
D. Penetrating keratoplasty
134. The color of fluorescein staining in corneal ulcer is:
A. Yellow
B. Blue
C. Green
D. Red
135. Following injury to the exiting eye, sympathetic ophthalmitis begins in:
A. In 4-6 days
B. In 4-6 weeks
C. 8-12 weeks
D. After 12 weeks
136. In intracapsular cataract extraction which part of the lens capsule is left behind?
A. Anterior lens capsule
B. Posterior lens capsule
C. Equatorial rim the lens capsule
D. No lens capsule is left behind
137. When there is intumescent cataract causing glaucoma?
A. Trephining should be done
B. Paracentesis is indicated
C. Lens should be extracted after control of IOP
D. Medical treatment only
138. Laser treatment in diabetic retinopathy is indicated in:
A. Dense vitreous haemorrhage
B. Diffuse macular oedema
C. Tractional retinal detachment
D. Macular epiretinal membrane distortion
139. In retinitis pigmentosa, the pigmentation in the retina starts at:
A. Posterior pole
B. Anterior to equator
C. Equator
D. At the disc
140. The incision size in phacoemulsification is:
A. 1 mm
B. 3 mm
C. 5 mm
D. 7 mm
141. Severe congenital ptosis with no levator function can be treated by:
A. Levator resection from skin side
B. Levator resection from conjunctival side
C. Fascia lata sling operation
D. Fasanella servat operation
142. In CRAO, a cherry red spot is due to :
A. Heamorrhage at macula
B. Increased choroidal perfusion
C. Increased in retinal perfusion at macula
D. The contrast between pale retina and reddish intact choriocapillaris
143. The most common primarily intraocular malignancy in adults is:
A. Retinoblastoma
B. Choroidal melanoma
C. Squamous cell carcinoma of conjunctiva
D. Iris naevus
144. A patient has an upper lid trichiasis with history of chronic eye irritation. The most
common causes are :
A. Stye
B. Trachoma
C. Infected chalazion
D. Spring catarrh
145. Chalazion is:
A. Acute suppurative inflammation of Meibomian glands
B. Chronic granulomatous inflammation of Meibomian glands
C. Retention cyst of the Meibomian glands
D. Neoplasm of the Meibomian glands
146. A patient complains of maceration of skin of the lids and conjunctival redness at
the inner and outer canthi. Conjunctival swab is expected to show:
A. Staphylococcus aureus
B. Staphylococcus viridans
C. Staphylococcus pneumonae
D. morax-Axenfield diplobacilli
147. Paralytic ectropion occurs in:
A. Third never paralysis
B. Trigeminal never paralysis
C. Facial never paralysis
D. Trochlear never paralysis
Answers
1-b 2-b 3-b 4-b 5-a 6-d
7-c 8-c 9-a 10-d 11-a 12-b
13-d 14-b 15-d 16-a 17-d 18-c
19-b 20-a 21-d 22-c 23-d 24-a
25-c 26-a 27-c 28-b 29-d 30-e
31-c 32-c 33-a 34-b 35-e 36-e
37-d 38-c 39-d 40-d 41-c 42-d
43-c 44-c 45-c 46-c 47-a 48-c
49-c 50-d 51-d 52-b 53-c 54-c
55-a 56-c 57-a 58-b 59-c 60-d
61-c 62-d 63-c 64-c 65-c 66-a
67-c 68-a 69-a 70-d 71-b 72-a
73-b 74-d 75-a 76-b 77-c 78-d
79-b 80-c 81-e 82-e 83-b 84-b
85-c 86-d 87-e 88-a 89-d 90-d
91-b 92-a 93-b 94-d 95-c 96-b
97-b 98-c 99-b 100-a 101-c 102-a
103-c 104-c 105-b 106-c 107-a 108-b
109-b 110-b 111-a 112-c 113-b 114-c
115-c 116-b 117-d 118-a 119-b 120-b
121-d 122-c 123-a 124-a 125-b 126-a
127-c 128-c 129-b 130-b 131-b 132-a
133-d 134-c 135-b 136-d 137-c 138-b
139-c 140-b 141-c 142-d 143-b 144-b
145-b 146-d 147-c
Slit lamp examination of your patient reveals bulging, centrally thinned corneas.
Refractometry shows an increase in astigmatism. The patient probably has:
A. flat cornea
B. keratoconjunctivitis
C. keratoconus
D. Exophthalmus

Anterior chamber is shallow in


A. buphthalmos
B. open angle glaucoma
C. closed angle glaucoma
D. Aphakia

Tremulousness of iris is seen in


A. chronic iridocyclitis
B. closed angle glaucoma
C. aphakia
D. senile immature cataract

White pupillary reflex is seen in


A. retinoblastoma
B. malignant melanoma
C Vitreous haemorrhage
D. Open angle glaucoma

The most accurate method of measuring IOP is


A. digital
B. applanation
C. air-puff non contact tonometer
D. Gonioscopy

A patient is presenting with ocular pain, positive fluorescein staining of the cornea
with a constricted pupil, and a high intra ocular pressure. The following medication is
contraindicated:
A. Atropine
B. Pilocaprine
C. Carbonic anhydrase inhibitors
D. Beta-blockers
Chronic use of topical steroids may lead to
A. iris atrophy
B. glaucoma
C. corneal opacity
D. Retinopathy

A young child suffering from fever and sore throat began to complain of
lacrimation. On examination, follicles were found in the lower palpebral conjunctiva
with tender preauricular lymph nodes. The most probable diagnosis is:
A. Trachoma
B. Staphylococcal conjunctivitis
C. Adenoviral conjunctivitis
D. Phlyctenular conjunctivitis

YAG laser is used in the treatment of


A. diabetic retinopathy
B. open angle glaucoma
C. after cataract
D. retinal detachment

Regarding buphthalmos, which is correct


A. shallow anterior chamber
B. bilateral
C. trabeculectomy is the treatment of choice
D. small corneal diameter (less than 10mm)

Treatment of choice in primary open angle glaucoma is


A. cyclodialysis
B. iridectomy
C. cyclodiathermy
D. Medical

The optic nerve extends up to


A. optical chiasma
B. optic tract
C. lateral geniculate body
D. optic radiation
'D' shaped pupil is seen in
A. iridocyclitis
B. iridodialysis
C. glaucoma
D. dislocation of lens

Hordeolum externum is an inflammation of


A. lid margin
B. tarsal plate
C. meibomian gland
D. Zeis gland

Neovascular glaucoma can be best treated by:


A. trabeculectomy
B. pilocarpine
C. panphotocoagulation of retina
D. timolol maleate

In myopia which one of the following is most common?


A. Equatorial staphyloma
B. Posterior staphyloma
C. Ciliary staphyloma
D. Intercalary staphyloma

Distichiasis is:
A. Misdirected eyelashes
B. Accessory row of eyelashes
C. Downward droping of upper lid
D. Outward protusion of lower lid

Arlt's line is present is:


A. upper palpebral conjunctiva
B. Lower palpebral conjunctiva
C. Bulbar conjunctiva
D. Cornea
The color of fluorescein staining in corneal ulcer is:
A. Yellow
B. Blue
C. Green
D. Red

Following injury to the exiting eye, sympathetic ophthalmitis begins in:


A. In 4-6 days
B. In 4-6 weeks
C. 8-12 weeks
D. After 12 weeks

In intracapsular cataract extraction which part of the lens capsule is left behind?
A. Anterior lens capsule
B. Posterior lens capsule
C. Equatorial rim the lens capsule
D. No lens capsule is left behind

Laser treatment in diabetic retinopathy is indicated in:


A. Dense vitreous haemorrhage
B. Diffuse macular oedema
C. Tractional retinal detachment
D. Macular epiretinal membrane distortion

In retinitis pigmentosa, the pigmentation in the retina starts at:


A. Posterior pole
B. Anterior to equator
C. Equator
D. At the disc

The incision size in phacoemulsification is:


A. 1 mm
B. 3 mm
C. 5 mm
D. 7 mm
In CRAO, a cherry red spot is due to :
A. Heamorrhage at macula
B. Increased choroidal perfusion
C. Increased in retinal perfusion at macula
D. The contrast between pale retina and reddish intact choriocapillaris

The most common primarily intraocular malignancy in adults is:


A. Retinoblastoma
B. Choroidal melanoma
C. Squamous cell carcinoma of conjunctiva
D. Iris naevus

Chalazion is:
A. Acute suppurative inflammation of Meibomian glands
B. Chronic granulomatous inflammation of Meibomian glands
C. Retention cyst of the Meibomian glands
D. Neoplasm of the Meibomian glands

You might also like